Está en la página 1de 52

500 Oncolgy Nursing Questionnaires

1. Mr. Smith has laryngeal carcinoma with regional


recurrence. During a routine physical exam you notice he a. 0.5-10%
is experiencing stridor. This is most likely an indication of b. 10-30%
which of the following? c. 30-50%
a. Extrathoracic airway obstruction d. 50-
b. Intrathoracic airway obstruction
c. Tumor
7. Early-stage prostate cancer commonly involves
d. a and c
surgery or radiation therapy. Diagnostic transurethral
resection of the prostate (TURP) commonly causes
2. Which of the following is the most accurate measure of
retrograde ejaculation and may be associated with erectile
dyspnea in the individual with cancer?
dysfunction what percentage of the time?
a. Respiratory rate
b. Oxygen saturation
a. 10%
c. Arterial Blood Gas level
b. 25%
d. Patient self-report
c. 35%
d. 50%
3. A patient who just been diagnosed with lung cancer
denies the diagnosis and refuses to hear about it. The
8. Chemotherapy agents damage the hair most when it is
patients denial behaviour is
in which phase of hair growth?
a. Adaptive
a. Anagen
b. Maladaptive
b. Catagen
c. Value neutral
c. Telogen
d. None of the above
d. Transitional
Which
4. Support of loss and grief in the person suffering from
9. Which of the following chemotheraphy agents is least
cancer is primarily concerned with which of the following?
likely to cause hair loss?
a. How to manage symptoms of cancer
a. Cyclophosphamide
b. How to increase masked grief
b. Docetaxel
c. How to identify and aware of losses
c. Vinorelbine
d. How to use denial to cope with impending loss
d. Etoposide

5. Which of the following symptoms is least diagnostic of


10. Which of the following is leading cause of cancer death
anxiety in the person with cancer?
in women in developing counties such as Thailand,
Vietnam and Columbia?
a. Anxiety
a. Ovarian Cancer
b. Distractibility
b. Cervical Cancer
c. Worry
c. Breast Cancer
d. Restlessness
d. Gastrointestinal Cancer

6. When standardized psychiatric interviews and


11. In the Asian culture illnesses such as cancer are
research diagnostic criteria are used, the prevalence of
believed to be due to which of the following?
anxiety among cancer patients is approximately which of
the following?

Fourth Year- BSN Page 1


500 Oncolgy Nursing Questionnaires

a. An imbalance between yin and yang a. The total self-appraisal of cancer patients, both men
b. A curse by a spirit/spiritual imbalance and women
c. An obstruction of chi (an essential life energy) b. The effects of various treatments on relationships of
d. All of the above cancer patients with significant others
c. Women with gynaecologic or breast cancer or males
12. Effective cancer control is influenced most by which of with testicular or prostate cancer
the following? d. The interaction of variables such as age, depression,
and activity status on the psychosocial aspects of social
a. Government policy health
b. Routine chest x-rays
c. Hygiene 17. Which of the following systemic treatments for breast
d. A low-fat di cancer is least likely to cause impairment in sexual
function?
13. Nonsteroidal anti-inflammatory drugs have been
purported to prevent which of the following? a. Monthly goserelin injection
b. Monthly goserelin injection plus tamoxifen
a. Bladder Cancer c. Chemotherapy alone
b. Breast Cancer d. Tamoxifen alone
c. Colorectal Cancer
d. Prostate Cancer
18. Two basic nursing interventions for alterations in
14. Which of the following is one of the key issues for sexual health encountered by cancer patients are:
adult children of cancer patients?
a. Disruption of current family relationships a. Education and counseling
b. Behavior problems b. Screening and role playing
c. Lack of involvement in decision making regarding the c. Affective therapy and role modelling
parents illness d. Enhancing reality surveillance and reinforcing
d. Assuming the protector role and shielding parents personal power
from discussing feelings
19. Sexuality in the cancer patient may be affected by the
15. Women treated for gynaecologic malignancy face a following factors exept
50% or greater chance of sexual dysfunction due primarily
to which of the following. a. Psychosexual changes associated with mutagenicity
b. Physiologic problems of fertility and sterility
a. Changes in body image c. Psychologic issues such as loss of self esteem and fears
b. Changes in sexual functioning of abandonment
c. Fertility issues d. Changes in body appearance resulting from therapy
d. All of the above
20. When documenting patterns of hair loss due to
16. Most empirical studies dealing with the relationship of chemotherapy, it is important to include all but which of
cancer to self-care concept have focused on the following assessment criteria
a. Family history of male pattern baldness
b. Lenght, texture, and curl/wave of hair

Fourth Year- BSN Page 2


500 Oncolgy Nursing Questionnaires

c. Condition of the scalp


d. Description of patterns of hair loss over entire body a. Medical and family history
b. Type of treatment and side effects experienced
c. cancer site and stage of disease
21. For patients who undergo surgery for gastrointestinal d. Severity or duration of disease
cancer, possible organic sexual dysfunction is most closely
associated with which of the following?
26. A 68-year-old woman is diagnosed with
thrombocytopenia due to acute lymphocytic leukemia. She
a. Placement of a colostomy is admitted to the hospital for treatment. The nurse should
b. Removal of rectal tissue assign the patient
(A) to a private room so she will not infect other patients
c. Changes in body image and health care workers.
d. Responses by family and friends (B) to a private room so she will not be infected by other
patients and health care workers.
(C) to a semiprivate room so she will have stimulation
22. The most frequently addressed factors contributing to
during her hospitalization.
cancer-related suicide or euthanasia are (D) to a semiprivate room so she will have the opportunity
to express her feelings about her illness.
a. Pain and other symptom distress
27. The nurse teaches a group of mothers of toddlers how
b. Advance illness and poor prognosis to prevent accidental poisoning. Which of the following
c. Family history of suicide or personal suicide history suggestions should the nurse give regarding medications?
d. Hopelessness and loss of self esteem or control (A) Lock all medications in a cabinet.
(B) Child proof all the caps to medication bottles.
(C) Store medications on the highest shelf in a cupboard.
23. Which of the following is least likely to be a cause of (D) Place medications in different containers.
anemia in the cancer patient?
28. While inserting a nasogastric tube, the nurse should
use which of the following protective measures?
a. Decreased red cell production (A) Gloves, gown, goggles, and surgical cap.
b. Iron deficiency (B) Sterile gloves, mask, plastic bags, and gown.
c. The primary disease process (C) Gloves, gown, mask, and goggles.
(D) Double gloves, goggles, mask, and surgical cap.
d. Radiation therapy
29. A 6-year-old boy is returned to his room following a
tonsillectomy. He remains sleepy from the anesthesia but
is easily awakened. The nurse should place the child in
which of the following positions?
24. A program that regards rehabilitation in cancer care
(A) Sims.
as a dynamic rather than a passive process is most likely to (B) Side-lying.
emphasize both ongoing reassessment and (C) Supine.
(D) Prone.
a. Customary convalescence 30. A nursing team consists of an RN, an LPN/LVN, and a
b. A hospital or community base nursing assistant. The nurse should assign which of the
c. Redefinition of goals following patients to the LPN/LVN?
(A) A 72-year-old patient with diabetes who requires a
d. Frequent nursing referrals
dressing change for a stasis ulcer.
(B) A 42-year-old patient with cancer of the bone
25. Which of the following factors have been found to be complaining of pain.
most closely related to rehabilitation needs of the cancer (C) A 55-year-old patient with terminal cancer being
transferred to hospice home care.
patient?

Fourth Year- BSN Page 3


500 Oncolgy Nursing Questionnaires

(D) A 23-year-old patient with a fracture of the right leg c) assess the client's mental status
who asks to use the urinal. d) elevate the client's head of the bed

31. Which of these findings in the breast of a patient who 37. The nurse on the oncology unit is planning care for
is suspected of having breast cancer would support the client with colon cancer who is refusing a diagnostic
the diagnosis? test. Which action is most appropriate for the nurse to take
a) complaints of dull, achy, pain first?
b) palpation of a mobile mass a) call the radiology department to let them know
c) presence of an inverted nipple the client will not be going to take the test
d) area of discoloration skin b) speak with the client to determine the reason for
refusing the test
c) inform the health care provider that the client is
32. A nurse is caring for a client with an internal refusing the test
radiation implant. Which of the followinginstructions is d) ask the client's spouse why the client is refusing the test
appropriate?
a) allow the client to go to the bathroom 38. A nurse is admitting a 63-year old male reporting
b) avoid creams and lotions hemoptysis and weight loss. The nurse identifies that the
c) visitors are allowed to stay in the room highest priority risk factor for lung cancer for this client is:
d) the client should remain in bed during the entire a) family history of lung cancer
duration of treatment b) the client works in a chemical factory
c) the client lives in a coal mining area
33. How often should a female who is above 40 years old, d) the client uses chewing tobacco
go for cancer detectionexamination?
a) daily 39. The nurse is caring for a client with a diagnosis of
b) weekly cancer who is immunosuppressed. The nurse would
c) monthly consider implementing neutropenic precautions if
d) yearly the client's white blood cell count was which of the
following?
34.The client is receiving internal radiation therapy. The a) 2,000 cells/mm3
nurse should b) 5,800 cells/mm3
a) remember to give the badge to the next-shift nurse c) 8,400 cells/mm3
b) maintain a 30-minute close contact with the patient in a d) 11,500 cells/mm3
shift
c) wear gloves, mask and gown when entering the client's 40.A nurse is caring for a child after removal of a brain
room tumor. The nurse assesses the child for which of the
d) instruct relatives no to visit the client during the entire following signs that would indicate that brainstem
duration of the treatment involvement occurred during the surgical procedure?
a) inability to swallow
35 A nurse is assessing a client with metastatic breast b) elevated temperature
cancer who reports nocturia, weakness, nausea and c) altered hearing ability
vomiting. The client's serum electrolytes include d) orthostatic hypotension
potassium 4.2 mEq/L, sodium 135 mEq/L, calcium 7.0
mEq/L, and magnesium 2.0 mEq/L. Based on the 41. The health education nurse provides instructions to a
assessment findings,the priority action for the nurse is to: group of clients regarding measures that will assist in
a) start client on fluid restriction preventing skin cancer. Which statement by
b) administer calcium gluconate a client indicates a need for further instructions?
c) increase the client's IV fluids a) I will avoid sun exposure after 3 pm
d) administer Allopurinol b) I will use sunscreen when participating in outdoor
activities
36.. The nurse on the oncology unit enters the room of the c) I will wear a hat, opaque clothing, and sunglasses when
client with lung cancer. Which action is most appropriate in the sun
for the nurse to do first? d) I will examine my body monthly for any lesions that
a) check the client's IV infusion pump and IV fluid rate may be suspicious
b) take the client's blood pressure and pulse

Fourth Year- BSN Page 4


500 Oncolgy Nursing Questionnaires

42.The client is undergoing radiation therapy to treat lung b) ambulation three times daily
cancer. Following treatment, the nurse notes erythema on c) monitoring the platelet count
the client's chest and neck, and the client is complaining of d) monitoring for pathological fractures
pain at the radiation site. The nurse interprets
this assessment data a(n): 48. The nurse is monitoring the laboratory results of
a) allergic reaction to the radiation a client preparing to receive chemotherapy. The nurse
b) superficial injury to tissue from the radiation determines that the white blood cell count is normal if
c) cutaneous reaction to products formed by the lysis of which of the following results were present?
the neoplastic cells a) 2000 to 5000 cells/mm3
d) ischemic injury, much like pressure ulcer formation. b) 3000 to 8000 cells/mm3
caused by pressure from the linearaccelerator c) 5000 to 10000 cells/mm3
d) 7000 to 15000 cells/mm3
43. The community nurse is conducting a health
promotion program at a local school and is discussing the 49. The community health nurse is instructing a group of
risk factors associated with cervical cancer. Which of the female clients about breast self-examination. The nurse
following, if identified by the client as a risk factor to instructs the clients to perform the examination:
cervical cancer, indicates a need for further teaching? a) at the onset of menstruation
a) smoking b) every month during ovulation
b) multiple sex partners c) weekly at the same time of day
c) first intercourse after age 20 d) 1 week after menstruation begins
d) annual gynecological examinations

44. The client is diagnosed as having a bowel tumor and 50. The nurse is caring for a client who has undergone
several diagnostic tests are prescribed. The nurse vaginal hysterectomy. The nurse avoids which of the
understands that which test will confirm the diagnosis of following in the care of this client?
malignancy? a) elevating the knee on the bed
a) biopsy of tumor b) assisting with range-of-motion leg exercises
b) abdominal ultrasound c) removal of antiembolism stockings twice a day
c) magnetic resonance imaging d) checking placement of pneumatic compression boots
d) computed tomography scan
20 51. The nurses assesses that the client with cancer is not
ready for teaching when the client asks:
45. The nurse is reviewing the laboratory results of 1. Am I going to loose my hair?
a client diagnosed with multiple myeloma. Which of the 2. Should I get a second opinion?
following would the nurse expect to note specifically in 3. Will this make me really sick?
this disorder? 4. Will I have to stop exercising at the gym?
a) increased calcium level
b) increased white blood cells 52. Knowing that chemotherapy affects the taste buds, the
c) decreased blood urea nitrogen level nurse would have the client
d) decreased number of plasma cells in the bone marrow 1. Increase the amount of spices in the food.
2. Avoid red meats.
46. The nurse is instructing the client to perform a 3. Medicate with Compazine before meals.
testicular self-examination. The nurse tells the client: 4. Eat foods that are hot in temperature.
a) to examine the testicles while lying down
b.)that the best time for the examination is after a shower
c) to gently feel the testicles with one finger to feel for a 53. In planning care for a client with a platelet count of
growth 8000 and a WBC of 8000 the nurse can expect to:
d) that testicular self-examinations should be done at least 1. Remove flowers from the room.
every 6 months 2. Encourage fruits and vegetables.
3. Use strict hand washing technique.
47. The client with cancer is receiving chemotherapy and 4. Take temperature frequently.
develops thrombocytopenia. The nurse identifies which
intervention as the highest priority in the nursing plan of 54. The nurse is teaching a client with a WBC of 2000.
care? Which statement the client makes indicates an
a) monitoring temperature understanding of the teaching?

Fourth Year- BSN Page 5


500 Oncolgy Nursing Questionnaires

1. I will eat fresh fruits and vegetables to avoid 61. A 28-year-old woman is diagnosed as having iron
constipation. deficiency anemia. Imferon IM is ordered. The nurse
2. I will stay away from my cat. administers it using the Z track technique. The primary
3. I will avoid crowded places. reason for administering Imferon via Z track is to:
4. I will wash all my fruits and vegetables before I eat 1. Prolong the action of the drug.
them. 2. Prevent staining of the skin.
3. Improve the absorption rate.
55. In evaluating the client with cancer what best indicates 4. Increase the speed of onset of action
that nutritional status is adequate?
1. Calorie intake 62. The nurse is discussing dietary sources of iron with a
2. Weight is stable client who has iron deficiency anemia. The nurse knows
3. Amount of nausea and vomiting the client can select a diet high in iron when she selects
4. Serum protein levels which menu?
1. Milkshake, hot dog, beets.
56. An adult client with newly diagnosed cancer says, Im 2. Beef steak, spinach, grape juice.
really afraid of dying. Whos going to take care of my 3. Chicken salad, green peas, coffee.
children? What is the best initial response for the nurse to 4. Macaroni and cheese, coleslaw, lemonade.
make?
1. What makes you think you are going to die? 63. A client with iron deficiency anemia is to take ferrous
2. How old are your children? sulfate. She returns to clinic in two weeks. Which
3. This must be a difficult time for you. assessment by the nurse indicates the client has NOT been
4. Most people with your kind of cancer live a long time. taking iron as ordered?
1. The clients cheeks are flushed.
57. A client with terminal cancer yells at the nurse and 2. The client reports having more energy.
says, I dont need your help. I can bathe myself. Which 3. The client complains of nausea.
stage of grief is the client most likely experiencing? 4. The clients stools are light brown.
1. Projection
2. Denial 64. A 66-year-old woman is being evaluated for pernicious
3. Anger anemia. What signs and symptoms would the nurse expect
4. Depression to assess in a client with pernicious anemia?
1. Easy bruising.
58. The nurse can expect a client with a platelet count of 2. Beefy red tongue.
8000 and WBC count of 8000 to be placed: 3. Fine red rash on the extremities.
1. In a private room. 4. Pruritus.
2. On protective isolation.
3. On bleeding precautions. 65. The nurse is caring for a client who is newly diagnosed
4. On neutropenic precautions. with pernicious anemia. The client asks why she must
receive vitamin shots. What is the best answer for the
59. Which statement the client makes indicates to the nurse to give?
nurse that the client understands external radiation? 1. Shots work faster than pills.
1. Ill stay away from small children since I am 2. Your body can not absorb Vitamin B12 from the
radioactive. stomach.
2. I wont wash these marks off until after my therapy. 3. Vitamins are necessary to make the blood cells.
3. Ill put lotion on my skin to keep it moist. 4. You can get more vitamins in a shot than a pill.
4. I will double flush the toilet each time I use the
bathroom. 66. A woman who has had pernicious anemia for several
years is seen in the clinic. She tells the nurse that she has a
60. When teaching and preparing a client for a bone tingling in her arms and legs. What question should the
marrow biopsy, the nurse would nurse ask initially?
1. Check for iodine allergy. 1. Has your activity level changed lately?
2. Position the client in fetal position with back curved. 2. Has your diet changed recently?
3. Have the client sign the consent form. 3. Have you been sitting more than usual?
4. Have the client remain NPO. 4. Have you been taking your medicine regularly?

Fourth Year- BSN Page 6


500 Oncolgy Nursing Questionnaires

67. A one-year-old is admitted to the hospital with sickle 3. Contaminated water.


cell anemia in crisis. Upon admission which therapy will 4. Intimate oral contact.
assume priority?
1. Fluid administration. 73. An 8-year-old is admitted to the unit with a diagnosis
2. Exchange transfusion. of acute lymphocytic leukemia. He was receiving a physical
3. Anticoagulant. exam prior to playing Little League baseball. Numerous
4. IM administration of iron and folic acid. ecchymotic areas were noted on his body. His mother
reported that he had been more tired than usual lately. The
68. The nurse is caring for a 15-month-old child who is childs mother says that he has had a cold for the last
newly diagnosed with sickle cell anemia. The mother asks several weeks. She asks if this is related to his leukemia.
why the child has not had any symptoms before now. The The nurses response is based on the knowledge that
nurses response is based on which knowledge? 1. Leukemia causes a decrease in the number of normal
1. Maternal antibodies have protected the child during the white blood cells in the body.
first year of life. 2. A chronic infection such as he has had predisposes a
2. Breast milk is a deterrent to sickle cell anemia. child to the development of leukemia.
3. The disease does not manifest until the child begins to 3. The virus responsible for colds has been implicated as a
walk. possible etiologic agent in leukemia.
4. Elevated fetal hemoglobin levels prevent sickling of red 4. Having an infection prior to the onset of leukemia is
cells. merely a coincidence.

69. Which statement is essential for the nurse to include in 74. A child who is receiving chemotherapy for leukemia
discharge teaching to the parents of a young child who has has stomatitis. Which of the following nursing care
sickle cell anemia? measures is essential?
1. Do not let her bump into things. She will bruise easily. 1. Using dental floss to clean the teeth.
2. Notify the physician immediately if she develops a fever. 2. Frequent cleaning of the mouth with an astringent
3. She will need special help with feeding. mouthwash.
4. Observe her frequently for difficulty breathing. 3. Use of an overbed cradle.
4. Swabbing the mouth with moistened cotton swabs.
70. The nurse has been teaching the mother of a child with
hemophilia about the care he will need. Which statement 75. A school age child is receiving chemotherapy for
the mother makes indicates a need for more instruction? leukemia. Which statement he makes indicates the best
1. If he needs something for pain or a fever, I will give him understanding and acceptance of what is happening to
acetaminophen instead of aspirin. him?
2. I will take him to the dentist for regular checkups. 1. I hope I wont loose my hair like the other kids.
3. I will keep him in the house most of the time. 2. See my new red hat. I like to wear it.
4. His medical identification bracelet arrived. 3. I want to go see my friend Harold who is in the hospital
with meningitis.
71. A 19-year-old college student reports to the health 4. When Im finished with the chemotherapy, the
service with a sore throat, malaise, and fever of four days leukemia will be gone forever.
duration. Examination shows cervical lymphadenopathy
and splenomegaly. Temperature is 103oF. Blood is positive
for heterophil antibody agglutination test. The nurse
knows that infectious mononucleosis is caused by 76. The nurse is taking the vital signs of the client admitted
1. Cytomegalovirus. with cancer of the pancreas. The nurse is aware that the
2. Beta hemolytic streptococcus. fifth vital sign is:
3. Epstein-Barr virus. a. Anorexia
4. Herpes simplex virus I. b. Pain
c. Insomia
72. A client who is diagnosed with infectious d. Fatigue
mononucleosis asks how he got this disease. The nurses
response is based on the knowledge that the usual mode of 77. The nurse is assessing the client admitted for possible
transmission is through oral cancer. The nurse identifies which of the following to
1. Contact with an open wound in the skin. be to a late-occuring sx. of oral cancer?
2. Genital contact. a. Warmth

Fourth Year- BSN Page 7


500 Oncolgy Nursing Questionnaires

b. Odor a. That cannot be assessed


c. Pain b. That is in situ
d. Ulcer with flat edges c. With increasing lymph node involvement
d. With distant metastasis
78 The nurse understand that the diagnosis of oral cancer
is confirmed with: 85. A client with cancer is to undergo an intravenous
a. Biopsy pyelogram. The nurse should:
b. Gram Stain a. Force fluids 24 hours before the procedure
c. Scrape cytology b. Ask the client to void immediately before the study
d. Oral washings for cytology c. Hold medication that effects the central nervous
system for 12 hours pre and post test
79. Before administering Methyterxate orally to the client d. Cover the client's reproductive organs with an x-ray
with cancer, the nurse should check the: shield.
a. IV site
b. Electrolytes 86. A removal of the left lower lobe of the lung is
c. Blood gases performed on a client with lung cancer. Which post-
d. Vital signs operative measure would usually be included in the plan?
a. Closed chest drainage
80. An elderly is dianosed with ovarian cancer. She has b. A tracheostomy
surgery followed by chemotherapywith a fluorouracil c. A mediastenal tube
(Adrucil) IV. What shoul the nurse do if she notices d. Percussion vibration and drainage
crystals in the IV medication?
a. Discard the solution and order a new bag 87. The nurse is caring for a client with laryngeal cancer.
b. Warm the solution Which finding ascertained in the health history would not
c. Continue the infusion and document the finding be common for this dx?
d. Discontinue the medication a. Foul breath
b. Dysphagia
81. The client with cancer refuses to care for herself. c. Diarrhea
Which action by the nurse would be best? d. Chronic Hiccups
a. Alternate nurses caring for the client so that the staff
will not get tired of caring for this client 88. A client has rectal cancer and is scheduled for an
b. Talk to the client and expalin the need for self-care abdominal perineal resection. What should be the priority
c. Explore the reason for the lack of motivation seen in nursing care during the post-op period?
the client a. Teaching how to irrigate the illeostomy
d. Talk to the doctor about the client's lack of motivation b. Stopping electrolyte loss in the incisional area
c. Encouraging a high-fiber diet
82. A client with cancer develosps xerostomia . The nurse d. Facilitating perineal wound drainage
can help alleviate the discomfort the client is experiencing
associated with xerostomia by: 89. A client is admitted with a Ewing's sarcoma. Which sx.
a. Offering hard candy would be expected due to this tumor's location?
b. Administering analgesic medications a. Hemiplegia
c. Splinting swollen joints b. Aphasia
d. Providing saliva substitute c. Nausea
83. The nurse notes the pt. care assisstant looking through d. Bone pain
the personal items of the client with cancer. Which action
should be taken by the registered nurse? 90. A client with cancer is admitted to the oncology unit.
a. Notify the police department as a robbery Stat lab values reveal Hgb 12.6, WBC 6500, k+ 1.9, uric acid
b. Report this behavior to the charge nurse 7.0, Na+ 136, and platelets 178,000. The nurse evaluates
c. Monitor the situation and note whether any items are that the client is experiencing which of the following?
missing a. Hypernatremia
d. Ignore the situation until items are reported missing b. Hypokalemia
c. Myelosuppression
84. The nurse is caring for a client with a malignancy. The d. Leukocytosis
classification of the primary tumor is Tis. The nurse should
plan care for a tumor. 91. A client has cancer of the liver. The nurse should be

Fourth Year- BSN Page 8


500 Oncolgy Nursing Questionnaires

most concerned d. Place the client's right arm across her body
a. Alteration in nutrition
b. Alteration in urinary elimination 98. A client receiving chemotherapy for breast cancer has
c. Alteration in skin integrity an order for Zofran (ondansetron) 8mg PO to be given 30
minutes before induction of the chemotherapy. The
92. The nurse on oncology is caring for a client with a purpose of the medication is to:
white blood count of 600. During evening visitation, a a. Prevent anemia
visitor brings a potted plant. What action should the nurse b. Promote relaxation
take? c. Prevent nausea
a. Allow the client to keep the plant d. Increase neutrophil counts
b. Place the plant by the window
c. Tell the family members to take the plant home 99. A nursing assisstant assigned to care for a client
d. Water the plant for the client receiving linear accelarator radium therapy for laryngeal
cancer states, "I don't want to be assigned to that
93 Which cancer is associated with an increased risk of radioactive pt." The nurse response by the nurse to:
colorectal cancer? a. Tell the nursing assisstant that the client is not
a. Low protein, complex carbohydrates radioactive
b. High protein, simple carbohydrates b. Tell the nursing assisstant to wear a radiation badge
c. High fat, refined carbohydrates to detect the amount of radiation that she is receiving
d. Low carbohydrates, complex protein c. Instruct her regarding the use of a lead-lined apron
d. Ask a co-worker to care for the client
94. A client has been hospitalized with a dx of laryngeal
cancer. Which factor is most significant in the development 100. A client with acute leukemia is admitted to the
of laryngeal cancer? oncology unit. Which of the following would be most
a. A family history of laryngeal cancer important for the nurse to inquire?
b. Chronic inhalation of noxious fumes a. "Have you noticed a change in sleeping habits
c. Frequent straining of the vocal cords recently?"
d. A history of alcohol and tobacco use b. "Have you had a respiratory infection in the last 6
months?"
95. A client is being treated for cancer with linear c. "Have you lost weight recently?"
acceleration radiation. The physician has marked the d. "Have you noticed changes in your alertness?"
radiation site with a blue marking pen. The nurse should:
a. Remove the unsightly markings with acetone or
alcohol 101. A 35 years old client with ovarian cancer is
b. Cover the radiation site with loose gauze dressing
prescribed hydroxyurea (Hydrea), an antimetabolite drug.
c. Sprinkle baby powder over the rediated area
d. Refrain from using soap or lotion on the marked area Antimetabolites are a diverse group of antineoplastic
agents that interfere with various metabolic actions of the
96. While caring for a client with cervical cancer, the nurse cell. The mechanism of action of antimetabolites interferes
notes that the radioactive implant is lying in the bed. The
nurse should: with:
a. Place the implant in a biohazard and bag return it to a. cell division or mitosis during the M phase of the cell
the lab cycle.
b. Give the client a pair of gloves and ask her to reinsert
b. normal cellular processes during the S phase of the cell
the implant
c. Use tongs to pick up the implant and return it to a cycle.
leadlined container c. the chemical structure of deoxyribonucleic acid (DNA)
d. Discard the implant in the commode and double- and chemical binding between DNA molecules (cell cycle
flush
nonspecific).
97. A client with breast cancer is returned to the room d. one or more stages of ribonucleic acid (RNA) synthesis,
following a right total mastectomy. The nurse should: DNA synthesis, or both (cell cyclenonspecific).
a. Elevate the client's right arm on pillows
b. Place the client's right arm in a dependent sling
c. Keep the client's right arm on the bed beside her

Fourth Year- BSN Page 9


500 Oncolgy Nursing Questionnaires

102. The ABCD method offers one way to assess skin d. Checking regularly for signs and symptoms of
lesions for possible skin cancer. What does the A stand for? stomatitis
a. Actinic
b. Asymmetry 107. What should a male client over age 52 do to help
c. Arcus ensure early identification of prostate cancer?
d. Assessment a. Have a digital rectal examination and prostate-specific
antigen (PSA) test done yearly.
103. When caring for a male client diagnosed with a b. Have a transrectal ultrasound every 5 years.
brain tumor of the parietal lobe, the nurse expects to c. Perform monthly testicular self-examinations,
assess: especially after age 50.
a. short-term memory impairment. d. Have a complete blood count (CBC) and blood urea
b. tactile agnosia. nitrogen (BUN) and creatinine levels checked yearly.
c. seizures.
d. contralateral homonymous hemianopia. 108. A male client complains of sporadic epigastric pain,
yellow skin, nausea, vomiting, weight loss, and fatigue.
104. A female client is undergoing tests for multiple Suspecting gallbladder disease, the physician orders a
myeloma. Diagnostic study findings in multiple myeloma diagnostic workup, which reveals gallbladder cancer.
include: Which nursing diagnosis may be appropriate for this
a. a decreased serum creatinine level. client?
b. hypocalcemia. a. Anticipatory grieving
c. Bence Jones protein in the urine. b. Impaired swallowing
d. a low serum protein level. c. Disturbed body image
d. Chronic low self-esteem
105.. A 35 years old client has been receiving
chemotherapy to treat cancer. Which assessment finding
suggests that the client has developed stomatitis 109. A male client is in isolation after receiving an
(inflammation of the mouth)? internal radioactive implant to treat cancer. Two hours
a. White, cottage cheeselike patches on the tongue later, the nurse discovers the implant in the bed linens.
b. Yellow tooth discoloration What should the nurse do first?
c. Red, open sores on the oral mucosa a. Stand as far away from the implant as possible and call
d. Rust-colored sputum for help.
b. Pick up the implant with long-handled forceps and
106. During chemotherapy, an oncology client has a place it in a lead-lined container.
nursing diagnosis of impaired oral mucous membrane c. Leave the room and notify the radiation therapy
related to decreased nutrition and immunosuppression department immediately.
secondary to the cytotoxic effects of chemotherapy. Which d. Put the implant back in place, using forceps and a
nursing intervention is most likely to decrease the pain of shield for self-protection, and call for help.
stomatitis?
a. Recommending that the client discontinue 101. Jeovina, with advanced breast cancer is prescribed
chemotherapy tamoxifen (Nolvadex). When teaching the client about this
b. Providing a solution of hydrogen peroxide and water drug, the nurse should emphasize the importance of
for use as a mouth rinse reporting which adverse reaction immediately?
c. Monitoring the clients platelet and leukocyte counts a. Vision changes

Fourth Year- BSN Page 10


500 Oncolgy Nursing Questionnaires

b. Hearing loss 115. A male client has an abnormal result on a


c. Headache Papanicolaou test. After admitting, he read his chart while
d. Anorexia the nurse was out of the room, the client asks what
dysplasia means. Which definition should the nurse
111.. A female client with cancer is being evaluated for provide?
possible metastasis. Which of the following is one of the a. Presence of completely undifferentiated tumor cells
most common metastasis sites for cancer cells? that dont resemble cells of the tissues of their origin
a. Liver b. Increase in the number of normal cells in a normal
b. Colon arrangement in a tissue or an organ
c. Reproductive tract c. Replacement of one type of fully differentiated cell by
d. White blood cells (WBCs) another in tissues where the second type normally isnt
found
112. A 34-year-old female client is requesting d. Alteration in the size, shape, and organization of
information about mammograms and breast cancer. She differentiated cells
isnt considered at high risk for breast cancer. What should
the nurse tell this client? 116. For a female client with newly diagnosed cancer, the
a. She should have had a baseline mammogram before nurse formulates a nursing diagnosis of Anxiety related to
age 30. the threat of death secondary to cancer diagnosis. Which
b. She should eat a low-fat diet to further decrease her expected outcome would be appropriate for this client?
risk of breast cancer. a. Client verbalizes feelings of anxiety.
c. She should perform breast self-examination during the b. Client doesnt guess at prognosis.
first 5 days of each menstrual cycle. c. Client uses any effective method to reduce tension.
d. When she begins having yearly mammograms, breast d. Client stops seeking information.
self-examinations will no longer be necessary.
113. Nurse Brian is developing a plan of care for marrow 117. A male client with a cerebellar brain tumor is
suppression, the major dose-limiting adverse reaction to admitted to an acute care facility. The nurse formulates a
floxuridine (FUDR). How long after drug administration nursing diagnosis of Risk for injury. Which related-to
does bone marrow suppression become noticeable? phrase should the nurse add to complete the nursing
a. 24 hours diagnosis statement?
b. 2 to 4 days a. Related to visual field deficits
c. 7 to 14 days b. Related to difficulty swallowing
d. 21 to 28 days c. Related to impaired balance
d. Related to psychomotor seizures
114. The nurse is preparing for a female client for
magnetic resonance imaging (MRI) to confirm or rule out a 118. A female client with cancer is scheduled for
spinal cord lesion. During the MRI scan, which of the radiation therapy. The nurse knows that radiation at any
following would pose a threat to the client? treatment site may cause a certain adverse effect.
a. The client lies still. Therefore, the nurse should prepare the client to expect:
b. The client asks questions. a. hair loss.
c. The client hears thumping sounds. b. stomatitis.
d. The client wears a watch and wedding band. c. fatigue.
d. vomiting.

Fourth Year- BSN Page 11


500 Oncolgy Nursing Questionnaires

119. Nurse April is teaching a client who suspects that c. Pregnancy complicated with eclampsia at age 27
she has a lump in her breast. The nurse instructs the client d. Human papillomavirus infection at age 32
that a diagnosis of breast cancer is confirmed by:
a. breast self-examination. 124.. A female client is receiving methotrexate (Mexate),
b. mammography. 12 g/m2 I.V., to treat osteogenic carcinoma. During
c. fine needle aspiration. methotrexate therapy, the nurse expects the client to
d. chest X-ray. receive which other drug to protect normal cells?
a. probenecid (Benemid)
120.. A male client undergoes a laryngectomy to treat b. cytarabine (ara-C, cytosine arabinoside [Cytosar-U])
laryngeal cancer. When teaching the client how to care for c. thioguanine (6-thioguanine, 6-TG)
the neck stoma, the nurse should include which d. leucovorin (citrovorum factor or folinic acid
instruction? [Wellcovorin])
a. Keep the stoma uncovered.
b. Keep the stoma dry. 125.. The nurse is interviewing a male client about his
c. Have a family member perform stoma care initially past medical history. Which preexisting condition may
until you get used to the procedure. lead the nurse to suspect that a client has colorectal
d. Keep the stoma moist. cancer?
a. Duodenal ulcers
121. A female client is receiving chemotherapy to treat b. Hemorrhoids
breast cancer. Which assessment finding indicates a fluid c. Weight gain
and electrolyte imbalance induced by chemotherapy? d. Polyps
a. Urine output of 400 ml in 8 hours
b. Serum potassium level of 3.6 mEq/L 126. Nurse Amy is speaking to a group of women
c. Blood pressure of 120/64 to 130/72 mm Hg about early detection of breast cancer. The average age of
d. Dry oral mucous membranes and cracked lips the women in the group is 47. Following the American
Cancer Society guidelines, the nurse should recommend
122. Nurse April is teaching a group of women to that the women:
perform breast self-examination. The nurse should explain a. perform breast self-examination annually.
that the purpose of performing the examination is to b. have a mammogram annually.
discover: c. have a hormonal receptor assay annually.
a. cancerous lumps. d. have a physician conduct a clinical examination every
b. areas of thickness or fullness. 2 years.
c. changes from previous self-examinations.
d. fibrocystic masses. 127. A male client with a nagging cough makes an
appointment to see the physician after reading that this
123.. A client, age 41, visits the gynecologist. After symptom is one of the seven warning signs of cancer. What
examining her, the physician suspects cervical cancer. The is another warning sign of cancer?
nurse reviews the clients history for risk factors for this a. Persistent nausea
disease. Which history finding is a risk factor for cervical b. Rash
cancer? c. Indigestion
a. Onset of sporadic sexual activity at age 17 d. Chronic ache or pain
b. Spontaneous abortion at age 19

Fourth Year- BSN Page 12


500 Oncolgy Nursing Questionnaires

128. For a female client newly diagnosed with radiation- c. Breast cancer requires a mastectomy.
induced thrombocytopenia, the nurse should include d. Men can develop breast cancer.
which intervention in the plan of care?
a. Administering aspirin if the temperature exceeds 102 132. Nurse Meredith is instructing a premenopausal
F (38.8 C) woman about breast self-examination. The nurse should
b. Inspecting the skin for petechiae once every shift tell the client to do her self-examination:
c. Providing for frequent rest periods a. at the end of her menstrual cycle.
d. Placing the client in strict isolation b. on the same day each month.
c. on the 1st day of the menstrual cycle.
129. Nurse Lucia is providing breast cancer education at d. immediately after her menstrual period.
a community facility. The American Cancer Society
recommends that women get mammograms: 133. Nurse Kent is teaching a male client to perform
a. yearly after age 40. monthly testicular self-examinations. Which of the
b. after the birth of the first child and every 2 years following points would be appropriate to make?
thereafter. a. Testicular cancer is a highly curable type of cancer.
c. after the first menstrual period and annually b. Testicular cancer is very difficult to diagnose.
thereafter. c. Testicular cancer is the number one cause of cancer
d. every 3 years between ages 20 and 40 and annually deaths in males.
thereafter. d. Testicular cancer is more common in older men.

130. Which intervention is appropriate for the nurse 134. Rhea, has malignant lymphoma. As part of her
caring for a male client in severe pain receiving a chemotherapy, the physician prescribes chlorambucil
continuous I.V. infusion of morphine? (Leukeran), 10 mg by mouth daily. When caring for the
a. Assisting with a naloxone challenge test before therapy client, the nurse teaches her about adverse reactions to
begins chlorambucil, such as alopecia. How soon after the first
b. Discontinuing the drug immediately if signs of administration of chlorambucil might this reaction occur?
dependence appear a. Immediately
c. Changing the administration route to P.O. if the client b. 1 week
can tolerate fluids c. 2 to 3 weeks
d. Obtaining baseline vital signs before administering the d. 1 month
first dose
135. A male client is receiving the cell cyclenonspecific
131. Nina, an oncology nurse educator is speaking to a alkylating agent thiotepa (Thioplex), 60 mg weekly for 4
womens group about breast cancer. Questions and weeks by bladder instillation as part of a
comments from the audience reveal a misunderstanding of chemotherapeutic regimen to treat bladder cancer. The
some aspects of the disease. Various members of the client asks the nurse how the drug works. How does
audience have made all of the following statements. Which thiotepa exert its therapeutic effects?
one is accurate? a. It interferes with deoxyribonucleic acid (DNA)
a. Mammography is the most reliable method for replication only.
detecting breast cancer. b. It interferes with ribonucleic acid (RNA) transcription
b. Breast cancer is the leading killer of women of only.
childbearing age. c. It interferes with DNA replication and RNA

Fourth Year- BSN Page 13


500 Oncolgy Nursing Questionnaires

transcription. b. Maintain an NPO status before the procedure


d. It destroys the cell membrane, causing lysis. c. Wear comfortable clothing and shoes for the
procedure
136. The nurse is instructing the 35 year old client to d. Drink six to eight glasses of water without voiding
perform a testicular self-examination. The nurse tells the before the test
client:
a. To examine the testicles while lying down 141. A male client is diagnosed as having a bowel tumor
b. That the best time for the examination is after a and several diagnostic tests are prescribed. The nurse
shower understands that which test will confirm the diagnosis of
c. To gently feel the testicle with one finger to feel for a malignancy?
growth a. Biopsy of the tumor
d. That testicular self-examination should be done at b. Abdominal ultrasound
least every 6 months c. Magnetic resonance imaging
d. Computerized tomography scan
137. A female client with cancer is receiving
chemotherapy and develops thrombocytopenia. The nurse 142. A female client diagnosed with multiple myeloma
identifies which intervention as the highest priority in the and the client asks the nurse about the diagnosis. The
nursing plan of care? nurse bases the response on which description of this
a. Monitoring temperature disorder?
b. Ambulation three times daily a. Altered red blood cell production
c. Monitoring the platelet count b. Altered production of lymph nodes
d. Monitoring for pathological fractures c. Malignant exacerbation in the number of leukocytes
d. Malignant proliferation of plasma cells within the bone
138. Gian, a community health nurse is instructing a
group of female clients about breast self-examination. The 143. Nurse Bea is reviewing the laboratory results of a
nurse instructs the client to perform the examination: client diagnosed with multiple myeloma. Which of the
a. At the onset of menstruation following would the nurse expect to note specifically in
b. Every month during ovulation this disorder?
c. Weekly at the same time of day a. Increased calcium
d. 1 week after menstruation begins b. Increased white blood cells
c. Decreased blood urea nitrogen level
139. Nurse Cecilia is caring for a client who has d. Decreased number of plasma cells in the bone marrow
undergone a vaginal hysterectomy. The nurse avoids
which of the following in the care of this client? 144. Vanessa, a community health nurse conducts a
a. Elevating the knee gatch on the bed health promotion program regarding testicular cancer to
b. Assisting with range-of-motion leg exercises community members. The nurse determines that further
c. Removal of antiembolism stockings twice daily information needs to be provided if a community member
d. Checking placement of pneumatic compression boots states that which of the following is a sign of testicular
cancer?
140. Mina, who is suspected of an ovarian tumor is a. Alopecia
scheduled for a pelvic ultrasound. The nurse provides b. Back pain
which preprocedure instruction to the client? c. Painless testicular swelling
a. Eat a light breakfast only d. Heavy sensation in the scrotum

Fourth Year- BSN Page 14


500 Oncolgy Nursing Questionnaires

b. Restrict fluid intake


145. The male client is receiving external radiation to the c. Teach the client and family about the need for hand
neck for cancer of the larynx. The most likely side effect to hygiene
be expected is: d. Insert an indwelling urinary catheter to prevent skin
a. Dyspnea breakdown
b. Diarrhea
c. Sore throat 150. The home health care nurse is caring for a male
d. Constipation client with cancer and the client is complaining of acute
pain. The appropriate nursing assessment of the clients
146. Nurse Joy is caring for a client with an internal pain would include which of the following?
radiation implant. When caring for the client, the nurse a. The clients pain rating
should observe which of the following principles? b. Nonverbal cues from the client
a. Limit the time with the client to 1 hour per shift c. The nurses impression of the clients pain
b. Do not allow pregnant women into the clients room d. Pain relief after appropriate nursing intervention
c. Remove the dosimeter badge when entering the
clients room
d. Individuals younger than 16 years old may be allowed
151 . A 25 year old patient is inquiring about the methods
to go in the room as long as they are 6 feet away from the or ways to detect cancer earlier. The nurse least likely
client identify this method by stating:
a. Annual chest x-ray.
b. Annual Pap smear for sexually active women only.
147. A cervical radiation implant is placed in the client
c. Annual digital rectal examination for persons over age
for treatment of cervical cancer. The nurse initiates what 40.
most appropriate activity order for this client? d. Yearly physical and blood examination
a. Bed rest
152. The removal of entire breast, pectoralis major and
b. Out of bed ad lib
minor muscles and neck lymph nodes which is followed by
c. Out of bed in a chair only skin grafting is a procedure called:
d. Ambulation to the bathroom only a. Simple mastectomy
b. Modified radical mastectomy
c. Radiation therapy
148. A female client is hospitalized for insertion of an d. Radical mastectomy
internal cervical radiation implant. While giving care, the
nurse finds the radiation implant in the bed. The initial 153. Chemotherapy is one of the therapeutic modalities
action by the nurse is to: for cancer. This treatment is contraindicated to which of
the following conditions?
a. Call the physician
a. Recent surgery
b. Reinsert the implant into the vagina immediately b. Pregnancy
c. Pick up the implant with gloved hands and flush it c. Bone marrow depression
down the toilet d. All of the above
d. Pick up the implant with long-handled forceps and
154. The nurse is preparing Cyclophosphamide
place it in a lead container. (Cytoxan). Safe handling of the drug should be
implemented to protect the nurse from injury. Which of
149. The nurse is caring for a female client experiencing the following action by the nurse should be corrected?
a. The nurse should wear mask and gloves.
neutropenia as a result of chemotherapy and develops a b. Air bubbles should be expelled on wet cotton.
plan of care for the client. The nurse plans to: c. Label the hanging IV bottle with ANTINEOPLASTIC
a. Restrict all visitors

Fourth Year- BSN Page 15


500 Oncolgy Nursing Questionnaires

CHEMOTHERAPY sign. a. Analgesics before meals


d. Vent vials after mixing.5 b. Saline rinses every 2 hours
c. Aspirin every 4 hours
155. Neoplasm can be classified as either benign or d. Bland diet
malignant. The following are characteristics of malignant
tumor apart from: 161. Skin reactions are common in radiation therapy.
a. Metastasis Nursing responsibilities on promoting skin integrity
b. Infiltrates surrounding tissues should be promoted apart from:
c. Encapsulated a. Avoiding the use of ointments, powders and lotion to
d. Poorly differentiated cells the area
b. Using soft cotton fabrics for clothing
156. On a clinic visit a client who has a relative with c. Washing the area with a mild soap and water and
cancer, is asking about the warning signs that may relate patting it dry not rubbing it.
to cancer. The nurse correctly identifies the warning signs d. Avoiding direct sunshine or cold.
of cancer by responding:
a. If a sore healing took a month or more to heal, cancer 162. Nausea and vomiting is an expected side effect of
should be suspected. chemotherapeutic drug use. Which of the following drug
b. Presence of dry cough is one of the warning signs of should be administered to a client on chemotherapy to
cancer. prevent nausea and vomiting?
c. A lump located only in the breast area may suggest the a. Metochlopramide (Metozol)
presence of cancer. b. Succimer (Chemet)
d. Sudden weight loss of unexplained etiology can be a c. Anastrazole (Arimidex)
warning sign of cancer. d. Busulfan (Myleran)

157. In staging and grading neoplasm TNM system is 163 Radiation protection is very important to implement
used. TNM stands for: when performing nursing procedures. hen the nurse is not
a. Time, neoplasm, mode of growth performing any nursing procedures what distance should
b. Tumor, node, metastasis be maintained fromthe client?
c. Tumor, neoplasm, mode of growth a. 1 feet
d. Time, node, metastasis b. 2 feet
c. 2.5 feet
158. Breast self examination (BSE) is one of the ways to d. 3 feet
detect breast cancer earlier. The nurse is conducting a
health teaching to female clients in a clinic. During 164. The following are teaching guidelines regarding
evaluation the clients are asked to state what they learned. radiation therapy except:
Which of the following statement made by a client needs a. The therapy is painless
further teaching about BSE? b. To promote safety, the client is assisted by therapy
a. BSE is done after menstruation. personnel while the machine is in operation.
b. BSE palpation is done by starting at the center going c. The client may communicate all his concerns or needs
to the periphery in a circular motion. or discomforts while the machine is operating.
c. BSE can be done in either supine or standing d. Safety precautions are necessary only during the time
position. of actual irradiation.
d. BSE should start from age 20.
165. Contact of client on radiation therapy should be
159. A client had undergone radiation therapy (external). limited only to how many minutes to promote safety of the
The expected side effects include the following apart from: therapy personnel?
a. Hair loss a. 1 minute
b. Ulceration of oral mucous membranes b. 3 minutes
c. Constipation c. 5 minutes
d. Headache d. 10 minutes

160. Nurse Janet is assigned in the oncology section of 166. Nurse Joy is caring for a client with an internal
the hospital. Which of the following orders should the radiation implant. When caring for the 1client, the nurse
nurse question if a client is on radiation therapy? should observe which of the following principles?

Fourth Year- BSN Page 16


500 Oncolgy Nursing Questionnaires

a. Limit the time with the client to 1 hour per shift c. Incision appearance
b. Do not allow pregnant women into the clients room d. Urine specific gravity
c. Remove the dosimeter badge when entering the
clients room 172. A male client is admitted to the hospital with a
d. Individuals younger than 16 years old may be allowed suspected diagnosis of Hodgkins disease. Which
to go in the room as long as they are 6 feet away from the assessment findings would the nurse expect to note
client specifically in the client?
a. Fatigue
167. A cervical radiation implant is placed in the client b. Weakness
for treatment of cervical cancer. The nurse initiates what c. Weight gain
most appropriate activity order for this client? d. Enlarged lymph nodes
a. Bed rest
b. Out of bed ad lib 173. During the admission assessment of a 35 year old
c. Out of bed in a chair only client with advanced ovarian cancer, the nurse recognizes
d. Ambulation to the bathroom only which symptom as typical of the disease?
a. Diarrhea
168. A female client is hospitalized for insertion of an b. Hypermenorrhea
internal cervical radiation implant. While giving care, the c. Abdominal bleeding
nurse finds the radiation implant in the bed. The initial d. Abdominal distention
action by the nurse is to:
a. Call the physician 174. Nurse Kate is reviewing the complications of
b. Reinsert the implant into the vagina immediately colonization with a client who has microinvasive cervical
c. Pick up the implant with gloved hands and flush it cancer. Which complication, if identified by the client,
down the toilet indicates a need for further teaching?
d. Pick up the implant with long-handled forceps and a. Infection
place it in a lead container. b. Hemorrhage
c. Cervical stenosis
169. The nurse is caring for a female client experiencing d. Ovarian perforation
neutropenia as a result of chemotherapy and develops a
plan of care for the client. The nurse plans to: 175. Mr. Miller has been diagnosed with bone cancer. You
a. Restrict all visitors know this type of cancer is classified as:
b. Restrict fluid intake a. sarcoma.
c. Teach the client and family about the need for hand b. lymphoma.
hygiene c. carcinoma.
d. Insert an indwelling urinary catheter to prevent skin d. melanoma.
breakdown
176 Drugs can cause adverse events in a patient. Bone
170. The home health care nurse is caring for a male marrow toxicity is one of the most frequent types of
client with cancer and the client is complaining of acute
drug-induced toxicity. The most serious form of bone
pain. The appropriate nursing assessment of the clients
pain would include which of the following? marrow toxicity is:
a. The clients pain rating A. aplastic anemia.
b. Nonverbal cues from the client B. thrombocytosis.
c. The nurses impression of the clients pain C. leukocytosis.
d. Pain relief after appropriate nursing intervention D. granulocytosis.

171. Nurse Mickey is caring for a client who is 177. . Which of the following adverse effects is associated
postoperative following a pelvic exenteration and the
with levothyroxine (Synthroid) therapy?
physician changes the clients diet from NPO status to clear
liquids. The nurse makes which priority assessment before A. tachycardia
administering the diet? B. bradycardia
a. Bowel sounds C. hypotension
b. Ability to ambulate D. constipation

Fourth Year- BSN Page 17


500 Oncolgy Nursing Questionnaires

178. The chemotherapeutic DNA alkylating agents such as 183. . Serotonin release stimulates vomiting following
nitrogen mustards are effective because they: chemotherapy. Therefore, serotonin antagonists are
A. cross-link DNA strands with covalent bonds effective in preventing and treating nausea and vomiting
between alkyl groups on the drug and guanine related to chemotherapy. An example of an effective
bases on DNA. serotonin antagonist antiemetic is:
B. have few, if any, side effects. A. ondansetron (Zofran).
C. are used to treat multiple types of cancer. B. fluoxetine (Prozac).
D. are cell cycle-specific agents. C. paroxetine (Paxil).
D. sertraline (Zoloft).
179. Hormonal agents are used to treat some cancers. An
example would be: 184.. Methotrexate, the most widely used antimetabolite in
A. thyroxine to treat thyroid cancer. cancer chemotherapy does not penetrate the central
B. ACTH to treat adrenal carcinoma. nervous system (CNS). To treat CNS disease this drug must
C. estrogen antagonists to treat breast cancer. be administered:
D. glucagon to treat pancreatic carcinoma. A. intravenously.
B. subcutaneously.
180. Chemotherapeutic agents often produce a C. intrathecally.
certain degree of myelosuppression including D. by inhalation.
leukopenia. Leukopenia does not present
immediately but is delayed several days to weeks 185. Methotrexate is a folate antagonist. It inhibits
because: enzymes required for DNA base synthesis. To prevent
A. the patient's hemoglobin and hematocrit are harm to normal cells, a fully activated form of folic acid
normal. known as leucovorin (folinic acid; citrovorum factor) can
B. red blood cells are affected first. be administered. Administration of leucovorin is known
C. folic acid levels are normal. as:
D. the current white cell count is not affected by A. induction therapy.
chemotherapy. B. consolidation therapy.
C. pulse therapy.
181. Currently, there is no way to prevent D. rescue therapy.
myelosuppression. However, there are medications
available to elicit a more rapid bone marrow recovery. An 186. Patients undergoing chemotherapy may also be given
example is: the drug allopurinol (Zyloprim, Aloprim). Allopurinol
A. Epoetin alfa (Epogen, Procrit). inhibits the synthesis of uric acid. Concomitant
B. Glucagon. administration of allopurinol prevents:
C. Fenofibrate (Tricor). A. myelosuppression.
D. Lamotrigine (Lamictal). B. gout and hyperuricemia.
C. pancytopenia.
182. . Estrogen antagonists are used to treat estrogen D. cancer cell growth and replication.
hormone-dependent cancer, such as breast carcinoma.
Androgen antagonists block testosterone stimulation of 187. Superficial bladder cancer can be treated by direct
androgen-dependent cancers. An example of an androgen- instillation of the antineoplastic antibiotic agent
dependent cancer would be: mitomycin (Mutamycin). This process is termed:
A. prostate cancer. A. intraventricular administration.
B. thyroid cancer. B. intravesical administration.
C. renal carcinoma. C. intravascular administration.
D. neuroblastoma. D. intrathecal administration.

Fourth Year- BSN Page 18


500 Oncolgy Nursing Questionnaires

188. The most common dose-limiting toxicity of D. efficacy.


chemotherapy is: 194. A patient has been receiving chemotherapy to shrink
A. nausea and vomiting. a thoracic tumor. Chemotherapeutic side effects have
B. bloody stools. made eating difficult. The date for surgical removal of his
C. myelosuppression. tumor is approaching. Why would the physician initiate
D. inability to ingest food orally due to stomatitis total parenteral nutrition (TPN) at this point?
and mucositis. A. He wouldn't. TPN will cause the tumor to grow.
B. To treat malnutrition and optimize the surgical
189. Chemotherapy induces vomiting by: outcome.
A. stimulating neuroreceptors in the medulla. C. Because the patient is depressed about his
B. inhibiting the release of catecholamines. cancer and the chemotherapy side effects.
C. autonomic instability. D. It is the standard of all surgeons to initiate TPN
D. irritating the gastric mucosa. prior to surgery.

190. Myeloablation using chemotherapeutic agents is 195. A high-fiber diet is thought to reduce the risk of colon
useful in cancer treatment because: cancer because it:
A. it destroys the myelocytes (muscle cells). A. absorbs water from the intestinal wall.
B. it reduces the size of the cancer tumor. B. promotes the excretion of bile.
C. after surgery, it reduces the amount of C. stops diarrhea.
chemotherapy needed. D. is low in kilocalories.
D. it destroys the bone marrow prior to transplant.
196.Pain tolerance in an elderly patient with cancer would:
191 . Anticipatory nausea and vomiting associated with A. stay the same.
chemotherapy occurs: B. be lowered.
A. within the first 24 hours after chemotherapy. C. be increased.
B. 1-5 days after chemotherapy. D. no effect on pain tolerance.
C. before chemotherapy administration.
D. while chemotherapy is being administered. 197. In administering NSAID adjunctive therapy to an
elderly client with cancer, the nurse must monitor:
A. BUN and creatinine.
192. Levothyroxine (Synthroid) is the drug of choice for B. creatinine and calcium.
thyroid replacement therapy in patients with C. Hgb and Hct.
hypothyroidism because: D. BUN and CFT.
A. it is chemically stable, nonallergenic, and can be
administered orally once a day. 198. When administering a narcotic with a non-narcotic to
B. it is available in a single 25mg tablet making relieve severe cancer pain, the nurse must remember:
dosing simple. A. this combination enhances pain relief.
C. it is not a prodrug. B. this combination treats pain both centrally and
D. it has a short half-life. peripherally.
C. the narcotic potentiates action of the non-
193. Your patient has been on a narcotic analgesic for narcotic.
chronic pain from cancer. The dose she has been receiving D. each drug works in its own right.
is no longer bringing about the same pain relief as it once
did. The patient asks you why the medicine doesn't work 199. A cancer patient has had a resurgence of severe acute
anymore. You explain to her about: pain. Which of the following routes of medication is most
A. tolerance. appropriate for this patient?
B. potency. A. oral administration NSAIDS
C. receptor agonists. B. oral administration of narcotics

Fourth Year- BSN Page 19


500 Oncolgy Nursing Questionnaires

C. rectal administration of NSAIDS replies that its breast cancer. Which type of cancer causes
D. injectable pain reliever the most deaths in women?

200Which of the following statements about the use of a. Breast cancer


antidepressants with pain relief in cancer pain is true? b. Lung cancer
A. Antidepressants have no effect. c. Brain cancer
B. Antidepressants enhance the effect of d. Colon and rectal cancer
analgesics.
C. Antidepressants decrease the effect of
analgesics.
204. Client complains of sporadic epigastric pain, yellow
D. Antidepressants promote more rapid excretion
skin, nausea, vomiting, weight loss and fatigue. Suspecting
of the medications.
gallbladder disease, the physian orders a diagnostic
201. The nurse is caring for a recently married, 29-year- workup, which reveals gallbladder cancer. Which nursing
old female client, who was diagnosed with acute diagnosis may be appropriate for this client?
lymphocytic leukemia (ALL). The client is preparing for an
a. Anticipatory grieving
allogencic bone marrow transplant. Which statement by
b. Impaired swallowing
the client demonstrates the understands the informed
c. Disturbed body image
consent she gave about the diagnosis and treatment?
d. Chronic low self -esteem
aI should be able to finally start a family after Im finished
205.What is the most important postoperative instruction
the chemo.
the nurse must give a client who has just returned from the
bI always had a good appetite, even with chemo I operating room after receiving a subarachnoid block?
shouldnt have to make any changes to my diet.
a. Avoid drinking liquids until the gag reflex
cIll have to remain in the hospital for about 3 months returns
after my transplant. b. Avoid eating milk products for 24 hours
c. Notify a nurse if you experience blood in your
dIll only need chemotherapy treatment before receiving urine
my bone marrow transplant. d. Remain supine for the time specified by the
physician
202. A client is scheduled to receive methotrexate (Folex).
0.625 mg/kg P.O daily, to treat malignant lymphoma. 206 A client with colon cancer requires a permanent
Before administering the drug, the nurse reviews the colostomy because of the tumor. After surgery, the client
clients medication history. Which drug might interact with must learn how to irrigate the colostomy. When irrigating,
methotrexate? how far into the stoma should the client insert the
lubricated catheter?
a.Digoxin (Lonoxin)

b.Theophylline (Slo-phyllin)
a. 0.25 to 0.5
c.Probenecid (Bennemid) b. 1 to 1.5
c. 2 to 4
d. Famotidine (Pepcid)
d. 5 to 7
203.A client with uterine cancer asks the nurse, Which is
207.During the breast examination , which finding more
the most common type of cancer in women? The nurse
strongly suggests that the client has breast cancer?

Fourth Year- BSN Page 20


500 Oncolgy Nursing Questionnaires

d. Men can develop breast cancer

a. Slight asymmetry of the breast 212.Which action, displayed by a grieving husband over
b. A fixed nodular mass with dimpling of the his dying wife, would cause the nurse to suggest
overlying skin counselling?
c. Bloody discharge from the nipples
d. Multiple firm, round, freely movable masses a. He takes out wedding pictures and
that change with the menstrual cycle memorabilia to show you
b. He refuses to acknowledge his wifes
208.After cancer chemotherapy, a client experiences family and blames them for the
nausea and vomiting. The nurse should assign highest clients current health problem
priority to which intervention? c. He has already planned his wifes
funeral
a. Serving small portions of bland food d. He is planning to give away his wifes
b. Encouraging rhythmic breathing exercise treasured items to family members
c. Administering metoclopramide (Reglan) and
Dexamethasone(Decadron) as prescribed 213.The physician orders cystoscopy and random biopsies
d. Withholding fluids for the first 4 to 6 hours of the bladder for a client who reports painless hematuria.
after chemotherapy administration Test results reveal carcinoma in the situ in several bladder
regions. To treat bladder cancer, the client will have a
209.A client receiving chemotherapy to treat breast series or intravesical instillations of bacillus Calmette-
cancer. Which assessment finding indicates a fluid and Guerin (BCG), administered 1 week apart, when teaching a
electrolyte imbalance induced by chemotherapy? client about BCG, the nurse should mention that this drug
commonly causes:

a. Renal calculi
a. Urine output of 400 ml in 8 hours
b. Hematuria
b. Serum potassium level of 3.6 mEq/L
c. Delayed ejaculation
c. Blood pressure of 120/64 to 130/72 mm Hg
d. Impotence
d. Dry oral mucus membrane and cracked lips
214.A client with suspected lung cancer is suspected lung
210Which nursing intervention is most appropriate for a
cancer is scheduled for thoracentesis as a part of the
client with multiple myeloma?
diagnostic workup. The nurse reviews the clients history
a. Monitoring respiratory status for conditions that might contra indicate this procedure.
b. Balancing rest and activity Which condition is contraindication for thoracentesis?
c. Restricting fluid intake
a. A seizure disorder
d. Preventing bone injury
b. Chronic obstructive pulmonary
211. An oncology nurse educator is speaking to a
disease
womens group about breast cancer. Questions
c. Anemia
and comments from the audience reveal a
d. A bleeding disorder
misunderstanding of the aspects of the disease .
Various members of the audience have made all 215.A client is receiving the cell cycle- nonspecific
of the following statements. Which one is alkylating agent thiootepa(Thioplex), 60 mg weekly for
accurate? weeks by the bladder instillation as part of a
a. Mammography is the most reliable method for chemotherapeutic regimen to treat bladder cancer. The
detecting breast cancer client asks the nurse how the drug works. How does
b. Breast cancer is the leading killer of women of thiotepa exert its therapeutic effects?
childbearing age
c. Breast cancer requires a mastectomy

Fourth Year- BSN Page 21


500 Oncolgy Nursing Questionnaires

a. It interferes with deoxyribonucleic acid (DNA) 219.After receiving chemotherapy for lung cancer a clients
replication only platelet count falls to 98 000/ul. What term should the
b. It interferes with ribonucleic acid (RNA) nurse used to describe this low platelet count:
transcription only
c. It interferes with DNA replication and RNA a. Anemia
transcription
bLeukopenia
d. It destroy the cell membrane, causing lysis
c. Thromcytopenia
216.A home care nurse assesses for disease complications
in a client with bone cancer. The nurse knows that cone dNeutrophenia
cancer may cause which electrolyte disturbances?
220.To combat the most common adverse effect of
a. Hyperkalemia chemotherapy , the nurse must administer an:
b. Hyperkalemia
c. Htperkalemia a. Antiemetic
d. Hyperkalemia b. Antimetabolite
c. Antibiotic
217.After being in remission from Hodgkins disease for 18 d. Anticoagulant
months, a client develops a fever of unknown origin. The
physician orders a blind liver biopsy to rule out advancing 221.A client is hoispitalized with cell carcinoma of the
Hodgkins disease and infection. Twenty four hours after lung. To manage severe pain , the physicians prescribed a
biopsy , the client has a fever, complains of severe continous IV infusion of morphine. Which formula should
abdominal pain and seems increasing confuse. The nurse the nurse use to check that the morphine dose is
suspects that this findings result from: appropriate for the client?

a. Bleeding in the liver caused by the liver a. 1 mg/ kg of body weight


biopsy b. 5 mg/ kg of body weight
b. Perforation of the colon caused by the liver c. 5 mg/ 70 kg of body weight
biopsy d. 10 mg/ 70 kg of body weight
c. An allergic reaction to the contrast media
used during the liver biopsy
d. Normal post-procedural pain with the chnag 222.The nurse is providing best cancer education at the
ewith the level of conscoiysness resulting community facility. The American cancer society
from the pre existing fever. recommends that women get mammograms:

218.After the physicians explain the risk and benifits of a aYearly after age 40
clinical trial to a client, the client agrees to participate.
b After the birth of the first child and every two years there
Later that day, the client request clarification of the
after.
process involved in the clinical trial. As the member of the
multi-disciplinary team, how should the nurse respond? C After the first menstrual period and annually there after.

a. Provide the information requested dEvery three years between ages 20 and 40 and annually
b. Encourage the client to withdarw from the trial there after.
c. Not provide the information because its bearing
in the scope of nursing practice. 223.The nurse is caring for a patient with a long term
d. Tell the client that the information should come central venous catheter. Which care principle is correct?
from the physician that first presented it to him
aUse clean technique when assessing the port with a
needle

Fourth Year- BSN Page 22


500 Oncolgy Nursing Questionnaires

b If the needle to comes contaminated before accessing the happen if I die before my children are grown up? The
port, clean the needle with povidone iodine solution. most appropriate response by the nurse is:
a. There is no way of knowing what the future
C Clean the port with an alcohol pad before administering holds.
IV fluid through the catheter b. No matter what happens, your children will
be fine.
224.A client has just begun treatment with busulfan, 4 mg
c. You sound like you are afraid.
by mouty daily, for chronic myelogenous leukemia. The
d. Dont worry until you know the results.
client receives busulfan until his WBC count falls to
229. Which of the following client statements indicates
between 10 000/mm3 and 25 000/mm3. Then the blood is
the need for further client-teaching about
stopped. When should treatment resume?
chemotherapy?
a. When the WBC falls to 5000 mm3 a. Chemotherapy is a local treatment.
b. When loss hair begins to grow back b. I may experience nausea, vomiting, diarrhea,
c. When the WBC rises to 50 000 mm3 and loss during the treatments.
d. When the client displays anemia c. I may receive both oral and IV medications.
225. Which of the following is characteristic of a benign d. I may receive a combination of drugs over
neoplasm? different periods of time.
a. Rapid growth b. invasive 230. Which of the following statement is true about
c. noncohesive d. well-defined borders cell-kill hypothesis?
a. It explains the bodys mechanism of
226. Tina Blackey was recently diagnosed with breast protection from carcinogens.
cancer. She asks the nurse what steps she can take to b. It forms the basis for selecting specific classes
boost her immune system. The best response by the of chemotherapeutic drugs.
nurse is: c. It accounts for the severity of side effects.
a. Refuse chemotherapy; it weakens the d. It is the reasons why several courses of
immune system. chemotherapy are given.
b. Learn techniques to relieve or manage stress 231. A example of a chemotherapy catheter that is
more effectively. tunneled under the skin on the chest into the sub-
c. Ask your doctor to prescribe an clavian vein is
antidepressants. a. Hickman b. peripherally inserted central
d. Research herbal remedies. catheter c. mediport d. port-a-cath
232. Assessing for arrhythmias, gallops, and congestive
227 Which of the following explanations should the heart failure is an especially important nursing
nurse utilize regarding the magnetic resonance intervention for a client receiving:
imaging test used to diagnose a suspected brain a. Vinblastin (velban)
tumor? b. Doxorubicin (adriamycin)
c. 5-fluorouracil (5-FU)
a. This test sound waves. d. Vincristine (oncovin)
b. Before this test, the physician will inject you
with a radiopaque dye and then films will be 233.Marlene Morrison is scheduled to start
taken. chemotherapy. Which statement by Mrs. Morrison
c. The machine for this is like a tube, and you indicates a need for further client-teaching?
will lie inside. It may make thumping noises.
d. You must drink a radioactive substance a. I will have to drink plenty of fluids each day.
before this test. b. visits from my school-aged grandchildren
228. Jean Lawrence is undergoing diagnostic testing as will help my spirits.
part of the staging process. She tells the nurse, I am so c. I will need extra rest during this period of
worried they will find cancer everywhere; what will time.

Fourth Year- BSN Page 23


500 Oncolgy Nursing Questionnaires

d. I may need extra help at home to clean and c. I should protect my skin from sun exposure
prepare meals. during treatments.
d. I will wash the skin in the treatment area
234.Shelley Nordant is scheduled for surgery to daily using soap and hot water.
remove a cancerous tumor. She remarks to the nurse,
Why have surgery? I am going to have chemotherapy 238. George Houng is receiving combination therapy
just same. The most appropriate response by the of alpha-interferon with another chemotherapeutic
nurse is to: agent. Which statement indicates that the client and
family further teaching from the nurse?
a. Tell Mrs. Nordant that surgery must be
performed a. I will observe my husband for any changes in
b. Encouraged her to confer with the surgeon his mental status.
and the oncologist about the surgery b. I understand how to give the drug
c. Tell her that the combination of surgery and subcutaneously.
radiation may lead to a cure c. I can have severe flulike symptoms while
d. Document her concerns and give her taking these drugs.
reassurance d. I will limit my fluid intake.

235.Which of the following statements best describes 239.Stuart Eggleton, a client with prostate cancer, is
brachytherapy? trembling and is pacing in his room. His blood
pressure and pulse are slightly elevated. Mr. Eggleton
a. Brachytherapy involves placing radioactive may be experiencing:
materials at some distance from the tumor
b. Brachytherapy involves placing radioactive a. Some type of drug reaction
materials directly into the tumor site b. Anxiety
c. Brachytherapy is a holistic, nontradional c. Body image disturbance
cancer treatment d. Fear of unknown
d. Brachythaerapy is also referred to as
teletherapy 240. Which of the following statements indicates that
the client has misunderstood client-teaching about
236.Which of the following nursing actions is most alopecia from chemotherapy?
appropriate when caring for a client with unsealed
implanted radioisotope? a. I can wear a colorful scarf or turban.
b. I have already bought a wig.
a. Wear an abdominal apron c. My neighbor will be driving me to the look
b. Have the other client in the room wear a good, feel better program.
monitoring device to measure exposure d. After the chemotherapy is finished, my hair
c. Pick up any dislodged implants found in the will grow back the same as it was before the
bed linen with your sterile gloved hand treatments.
d. Dispose of clients body fluids in specially
marked containers 241. The partner of a client with cancer has been provided
with guidelines regarding when to call for help. Which of
237 Which of the following statement indicates that the following statements by the clients partner indicates
the client-teaching prior to an external radiation was that further teaching about the guidelines is needed?
successful?
a. Ill contact the physician if I see any new
a. if I have pain at my treatment site, I will bleeding from her rectum.
apply an ice pack. b. If her oral temperature is 100 F, Ill notify the
b. I will not have any intimate sexual contact physician.
while I am receiving treatments.

Fourth Year- BSN Page 24


500 Oncolgy Nursing Questionnaires

c. If she becomes withdrawn and cries b. If you would like to do this, you should
frequently, Ill notify the physician. try to include Mr. Rodriguez in the
d. Ill contact the physician if I notice a change planning.
in her eating patterns. c. Dont give up hope, he may still respond
to therapy.
242. Which of the following nursing assessments would d. You should wait until Mr. Rodriguez dies
indicate an improvement in a cancer client with nursing before making funeral plans.
diagnosis altered nutrition: less than body requirements?
246. Which strategy would not be recommended to a
a. Serum albumin less than 3.0 cancer client with problems eating?
b. A PPD response of more than 5 mm of
induration a. Eat icy- cold foods c. eat
c. A decrease in urinary creatinine level semisoft and liquid foods
d. An anthropometric measurement of 75% of b. Eat frequent, small meals d. eat
the standard spicy, warm foods

243. Which of the following nursing interventions would 247. Dana Biencarz has lung cancer and is experiencing
be most helpful in managing the nausea and vomiting of a facial and arm edema. The nurse should suspect that
client with cancer? Mrs.Biencarz has:

a. Instruct the client to drink plenty of fluids a. Allergic drug reaction


with each meal c. sepsis
b. Instruct the client to lie down for an hour b. Pericardial effusion d.
after eating superior vena cava syndrome
c. Encourage the client to eat dry foods
d. Instruct the client to follow a high-fat food 248. Rita Lopez has metastatic breast cancer and is
plan admitted to the hospital for pain management. During the
night she becomes incontinent, which has never occurred
244. Which of the following nursing actions is before. The appropriate action by the nurse is to:
contraindicated for the client with leukemia?
a. Recognize that the incontinence occurred as a
a. Assess the clients oral mucous membranes result of over sedation
daily b. Notify the physician that spinal cord
b. Instruct the client to brush and floss the teeth compression is suspected
three ties per day c. Realize that incontinence can occur in
c. Prevent dry mouth by using moisturizing metastatic bone disease
agents such as blistex d. Wait until the morning to see if the
d. Teach client to use smooth rather than sharp incontinence recur
utensils for eating
249..The nurse is caring for a client who has just a
245. The doctor has told Javier Rodriguez and his family modified radical mastectomy with immediate
that he has about 6 to 9 months to live. The family asks the reconstruction. Shes in her 30s and has two
nurse about making funeral arrangements ahead of time. young children. Although she worried abot her
The most appropriate response by the nurse is: future, she seems to be adjusting well to her
diagnosis what should the nurse do to support her
a. Many people do make arrangements coping?
ahead, but you might upset Mr. Rodriguez
if you tell him about the plans. a. Tell the client spouse or partner to be
supportive while she recover

Fourth Year- BSN Page 25


500 Oncolgy Nursing Questionnaires

b. Encourage the client to procede with the A. coughing deeply to prevent pneumonia
next phase of treatment B. Bending from the waist to prevent
c. Recommend that the client remain contracture deformity
cheerful for the sake of her children C. Remainins in a lying down supine position for
d. Refer the client to the American cancer 48h after surgery
society reach for recovery program or D. Deep breathing to promote lung expansion
another support program.
255. What is the highest priority, in terms of goal of care,
by the nurse after a clients prostatectomy?

250. Which lab test is it most important to monitor for a A. To prevent infection
client with lung cancer that has ectopic sites? B. To prevent bleeding
C. To prevent impotence
a. T3 b. T4 c. serum glucose D. To prevent respiratory distress
d. blood culture
SITUATION; A nurse is providing care to clients with
SITUATION; Nurse Tin-Tin is entrusted with the task of acquired immune deficiency syndrome.
promoting nursing care in client with various surgical
conditions. 256. The treatment of AIDS involves a palliative approach.
what does this approach means?
251. Physical preparation for a client schedule for
cystectomy and ilial conduit includes the administration A. Curing the disease
of? B. Relieving the clients discomfort
C. Providing rehabilitative support to the clients
A. Intravenous vitamins C family
B. Neomycin sulfate D. Promoting recovery
C. Bladder analgesic
D. Urinary antibiotic 257. A positive diagnosis for HIV infection is made based
on the result of which of the following test?
252. Which of the following methods of assessment is
correct to assess distention in a post operative client? A. ELISA
B. Western blot
A. Palpate the perineal area C. WBC count
B. Palpate the epigastric area D. CD4 cell count
C. Percuss and palpate the lower abdomen E. A and C
D. Percuss and palpate the upper abdomen F. A and B
G. A,B and C
253. After thyroidectomy, which of the following
H. A, B and D
intervention is the highest priority of the nurse?
258. Which principles of isolation should the nurse
A. Monitor the clients vital signs every 4h.
implement when obtaining the blood pressure of a client
B. Observe the dressing at the back of the neck
who has AIDS?
for the presence of blood
C. press gently around the incision to assess for A. Use of mask
subcutaneous emphysema B. Wearing clean gloves
D. positon the client laterally to promote C. Hand washing
drainage of secretions from the mouth D. Use of clean gown

254. Following cataract extraction, which client activity is 259. Which of the following practices limits a clients
allowed? exposure to thehuman immunodeficiency virus?

Fourth Year- BSN Page 26


500 Oncolgy Nursing Questionnaires

A. Using a condom during sexual intercourse C. Safety


B. Limiting sexual contact only to the spouse D. Fluids
C. Donating 200ml of blood
D. Having sexual intercourse only to those 265. To facilitate drainage of secretions from the operative
without HIV antibodies site, the nurse should?

260. Which of the following symptoms are most commonly A. Turn the client to the unoperative side every
seen in clients with AIDS? 2-3h
B. Turn the client to the operative side every 2-3
A. Low-grade fever and persistent diarrhea h
B. Cough and abdominal pain C. Keep the client supine until wound healing
C. Projectile vomiting and jaundice D. Place the client on semi-fowlers position at
D. Headache and rash least once every shift

SITUATION; The nurse is expected to perform her function SITUATION; Mr. Katrina Co, R.N., is assigned to take care of
in the operating room with outmost care. clients with traumatic injuries,electrolyte disturbances and
adrenal tumor.
261. A client is to undergo an exploratory laparotomy for
vague abdominal discomforts that his lead to abdominal 266. A client with chest trauma has a chest tube in place.
rigidity and severe pain. As the surgeon opens the clients after the removal of the tube, the clients PaO2 value is
abdomen, which of the following instruments shoul the 85mmHg. The nurse should interpret this value to indicate
nurse anticipate the surgeon to use? that the oxygen level?

A. Kocher A. Is normal
B. Mixtner B. Need to be increased
C. Kelly C. Need to be reported to the physician
D. Army navy immediately
D. Indicates that the client needs to keep breath
262. Surgical closure of the skin surfaces usually involes and cough
the nurse of which type of needle?
267. Which of the following methods is appropriate to
A. Straight needle documents the amount of drainage from a clients chest
B. Curved needle tube?
C. Rounded needle
D. eye less needle A. Aspirate the contents of the pleur evac and
measure the drainage
263. The surgeon closes the clients abdominal tissue layer. B. Empty the pleur evac during every shift
The nurse knows that closure of the abdominal tissue C. Mark the fluid level by placing a tape on the
layers begins with the peritoneum, extra peritoneal fat, collection chamber of the pleur evac
transversalis facia,followed by? D. Use a new set of pleur evac every shift

A. Deep facia, muscle subcutaneous tissue ,skin 268. Pulmonary capillary wedge measure is an indirect
B. Muscle, deep facia, subcutaneous tissue, skin measure of left ventricular end diastolic pressure. This
C. Subcutaneous tissue, deep facia, muscle, skin provides data on ventricular contractility. which catheter
D. Skin, muscle, subcutaneous tissue deep facia is appropriate to measure PCWP?

264. Following cataract surgery, which aspect of nursing A. Swan Ganz


care is the highest priority of the nurse? B. CVP
C. Tenckhoff
A. Nutrition
D. Grosshong
B. Airway

Fourth Year- BSN Page 27


500 Oncolgy Nursing Questionnaires

269. Which electrolyte imbalance occurs in a client with 274. To promote hemostasis in a client with transurethral
acute renal failure? resection of the prostate the clients retention catheter is
secured?
A. Hypocalcemia and hypophosphatemia
B. Hypomagnesemia and hypocalcemia A. To the lower abdomen
C. Hypomagnesemia and hypophophostemia B. To the leg
D. Hyponatremia and hyperkalemia C. To the bed
D. To the intravenous pole
270. Which of the following interventions take priority in a
client with pheochromocytoma? 275. Which of the following manifestations should the
nurse instruct the client to report to the physician
A. Counting the respiratory rate for one full immediately after undergoing TURP
minute
B. Monitoring the blood pressure A. Burning on urination
C. measuring the urine output in 24h B. Decreased urinary stream
D. Obtaining the axillary temperature every hour C. Pink- tinged urine on initial voiding
D. Urine output of 30ml/h
SITUATION; Care of client with TURP.
SITUATION: Mr. Jeremy Pontanes, R.N., is entrusted with
271. To promote bladder decompression and to prevent the task of promoting nursing care in client with
bladder decompression after transurethral resection of the disturbance in the cardiovascular system.
prostate, which intervention should be the nurses
priority? 276. Which of the following characteristic of pain will help
differentiate angina pectoris from myocardial infarction in
A. Monitoring the client for signs of hemorrhage terms of management?
B. Maintaining patency of the cystostomy tube
C. Assessing the client for signs of infection A. Duration
D. Ensuring patency of the continuous bladder B. Location
irrigation C. Severity
D. Cause
272. How should the nurse determine the urine outputof a
client on continuous bladder irrigation? 277. Which of the following is an early sign of heart
failure?
A. Deduct the amount of the irrigation solution
instilled from the total amount of output A. Neck vein distention
B. Deduct the amount of IV fluid infusion from B. Ankle edema
the total fluid intake of the client C. Dsypnea
C. Add the amount of all fluid intake then deduct D. Hepatomegaly
all the amount of output
D. Measure amount of output from the bladder 278. In a client with myocardial infarction, activity is
irrigation gradually increased based on which of the following
factors?
273. Which of the following is most common complication
of transurethral resection of the prostate in the early post A. Amount of weight loss
operative period. B. Severity of dsypnea
C. Degree of edema
A. Impotence D. Presence of absence of cyanosis
B. Infection
C. Bleeding 279. A client with history of myocardial infarction was
D. Urinary retention admitted due to dsypnea. The serum potassium level is

Fourth Year- BSN Page 28


500 Oncolgy Nursing Questionnaires

2.6mEq/L. Which of the following doctors ordere should A. Check the tube placement by instilling 10ml of
the nurse question? water into the tube
B. Change the tube every 24-48h.
A. Propanolol C. Ensure that the tube is connected to
B. Furosemide continuous suction
D. Assess the client for abdominal cramps and
C. Morphine
decreased bowel sounds.
D. Oxygen 1-21/min
SITUATION; Nurse Madonna is entrusted with the task of
280. The client is placed on a moderate sodium restricted promoting nursing care in client with disturbances in the
diet, which of the following food is allowed? respiratory system.
A. French fries 286. When a chest tube is being removed, the nurse should
B. Ice cream instruct the client to?
C. Catsup
D. Apples A. Cough vigorously
B. Deep breath
SITUATION; Nurse Henry is entrusted with the task of C. Inhale and hold breath
promoting nursing care in client with disturbances in the D. Exhale deeply
gastrointestinal tract.
287. Which the following equipment does now interfere
281. Cholangiography allows for the visualization of? with eating or talking and can best deliver oxygen in a
hypoxic client.?
A. Pancreas and large intestine
B. Liver and gallbladder A. Oxygen tent
C. Pancreatic duct and hepatic duct B. Oxygen mask
D. Common bile duct and hepatic duct C. Nasal cannula
D. Endotracheal tube
282. Which of the following beverages should the nurse
instruct a client who has been hospitalized due to peptic 288. Which reading of pulmonary capillary wedge
ulcer disease to avoid? pressure (in mmHg) needs to be reported to the physician
immediately?
A. Mineral water
B. Apple juice A. 5-10
C. Wine B. 15-20
D. Milk C. 25-30
D. 35-40
283. The most serious complication of the use of a
nasogastric tube is? 289. Which of the following client is at most risk for
tuberculosis?
A. Fluid and electrolyte imbalance
B. Ulceration A. A 35 year old newly married
C. Aspiration pneumonia B. A 68-year old client
D. Constipation C. A 12 year-old child
D. A 17 year-old adolescent
284. Which statement made by a client with ulcer indicates
an understanding of the need for ulcer medications? 290. Which of the following statement best describes the
primary purpose of pursed-lip breathing?
A. Anatacids can be taken with other drugs
B. Tagament will decrease the acid in my A. increase oxygen inhalation
stomach B. Maximize diaphragmatic extension
C. Maalox will coat my stomach C. Facilitate inhalation of carbon dioxide
D. I can take my antacids with aspirin D. Minimize the use of intercostals muscles
285. Which nursing action is of the highest priority in a SITUATION; Mr. Ronald is entrusted with the task of
client with nasogastric tube after gastric surgery? promoting nursing care in client with disturbances in the
biliary system.

Fourth Year- BSN Page 29


500 Oncolgy Nursing Questionnaires

291. Which assessment finding best indicates 297. Which of the following procedure for using an
deterioration of liver function? incentive spirometer is appropriate for the nurse to
provide the client with during pre operative instructions?
A. Anorexia
B. Constipation A. Inhale completely and exhale rapidly through
C. Weight gain the spirometer
D. Lethargy B. Exhale completely take slow deep breath
through the spirometer, hold breath then
292. Which of the following aspect of care should have the exhale through purse lip
highest priority for a client admitted due to cholecystitis? C. Inhale completely through the spirometer,
exhale and hold breath
A. Comfort D. Exhale through the spirometer, hold breath
B. Hydration then inhale
C. Health instruction
D. Obtaining thorough history 298. The stage of surgical anesthesia proceeds in which of
the following order?
293. Which of the following laboratory values is NOT
expected in a client with pancreatitis? A. Excitement, surgical anesthesia,loss of
consciousness and danger stage
A. Serum magnesium 3.2mEq/L B. Loss of consciousness, excitement, surgical
B. Serum glucose 45mg/dl anesthesia and danger stage
C. Serum potassium 5.5mEq/L C. Surgical anesthesia, excitement, loss of
D. Serum calcium 8mg/dl consciousness and danger stage
D. Loss of consciousness, surgical anesthesia,
294. Which client is the best candidate for the use of
excitement and danger stage
insulin pump?
299. What common side effect should the nurse monitor in
A. A 42 year-old businessman
a client who just received epidural anesthesia?
B. A 35 year-old working single mother
C. A 12 year-old mentally retarded A. Fever
D. An 82 year-old cognitively impaired B. Bradycardia
C. Hypotension
295. Which statement by a client with ulcer indicates
D. Pallor
understanding of the medication therapy?
300. Which of the following complications can result from
A. Antacids can be taken with other drugs
the use of general anesthesia during surgery?
B. Tagamet will decrease the acid in my stomach
C. Maalox will coat my stomach A. Fluid loss
D. I can take my antacid with asprin B. Peripheral retention
C. Atelectasis
SITUATION; Nurse jethro is entrusted with the task of
D. Muscle rigidity
promoting nursing care in client undergoing surgery.
SITUATION; Nurse Santini is taking care of Jay, a client
296. To decrease the anxiety of the client and his/her
family about an impending surgery, the nurse should? who underwent nephrectomy.

A. Describe the details of the surgery to the 301. Which of the following observation is expected in a
client and family post nephrectomy client?
B. Provide general information and answer
appropriate question of the client and family A. Urine output is 20ml/h
C. Empower the client to make decision by B. Drak red urine on the third post operative day
obtaining the clients consent C. Urine specific gravity: 1.003
D. Give full reassurance that the surgical team is D. Urinary drainage from the surgical wound a
experienced to handle the surgery
week after surgery

Fourth Year- BSN Page 30


500 Oncolgy Nursing Questionnaires

302. An appendectomy is usually performed on a client for C. Improve ventilation


kidney transplant to provide adequate space for the D. Maintain acid based balance
transplanted kidney, which is usually placed in the clients.
308. Which of the following foods should the nurse
A. Left lower quadrant encourage the client with tuberculosis and osteoporosis to
B. Right lower quadrant include in the diet?
C. Right upper quadrant
D. Left lower quadrant A. Soft drinks
B. Orange juice
303. Hourly monitoring of the clients urine in essential C. Beans
following nephrectomy to assess a common signs of kidney D. Yogurt
transplant rejection which is?
309. When communicating with the client with asthma and
A. abdominal pain Wernickes aphasia, the nurse should?
B. decreased urine output
C. Decreased blood pressure A. Provide adequate time for the client to
D. Anemia respond
B. Face the client and speak loudly
304. Which of the following laboratory data indicates the C. Use short and simple words and sentences
adverse effect of anti kidney rejection medication D. Provide a pencil and writing pad for the client
administered to a client
310. While performing respiratory assessment among
A. Platelet count 200,00/mm3 elderly clients, the nurse was able to obtain cues and
B. WBC count 3,800/mm3 possible cases of elderly abuse. Which client is most at risk
C. RBC count 5.1 million/mm3 for elderly abuse?
D. Hemoglobin 14mg/dL
A. A 68 year-old cognitively impaired male living
305. Which of the following laboratory data is the most alone
accurate blood test for determining dehydration? B. A 72 year-old female living with her spouse
and children
A. Urine specific gravity C. A 65 year-old male living with the household
B. Urine osmolarity help
C. Creatinine clearance D. A 67 year-old female living on her own
D. BUN
SITUATION; The following question pertain to clients with
SITUATION: Care of client with respiratory disorders. tubes.

306. Oseltamivir (tamiflu) is most effective if Tken within 311. A client just underwent choledoccholithotomy and
how many hours after the onset of the symptoms off has a T-tube in place. She calls the nurse and complains
influenza AH1N1? that the skin around the T-tube is excruciated. The nurse
should assess the skin and?
A. 48 h
B. 72h A. Reinforce the dressing
C. 96h B. Apply antiseptic solution
D. 100h C. Change the dressing
D. Use a skin barrier around the t-tube site
307. The priority goal of therapy for a client with chronic
obstructive pulmonary disease is to? 312. To promote continuous and proper functioning of
chest tube attached to a client, the nurse should keep the
A. Limit fluid administration
collection device?
B. Provides higher concentration of oxygen

Fourth Year- BSN Page 31


500 Oncolgy Nursing Questionnaires

A. Above the clients chest C. The amount of oxygen admitted is computed


B. Beside the client based on the clients weight
C. Below the level of the clients chest D. It is better if we try it for a few minutes first
D. At floor level
317. The nurse is providing pre-operative instruction in
313. After receiving the end shift report that a clients T- diaphragmatic breathing to a client with COPD. an
tube has a total drainage of 120 ml in 24h, which of the indication that the client understand the nurses
following intervention is the highest priority of the nurse? instruction is that he?

A. Refer the situation to the physician A. Cough before breathing


immediately B. Inhale through the nose and hold breath for 1-
B. document the amount on the clients kardex 2s before he exhale
C. Check the tube for the kinks and proper C. Exhale completely before increasing the rate
placement of the drainage bag of breathing
D. Empty the drainage bag and observe the client D. Inhale through the mouth
further.
318. In a client with COPD, cor pulmonale can possibly
314. Following the surgical creation of a permanent result in the enlargement of which of the following organs?
colostomy, the client call the nurse and ask why there is no
drainage from the colostomy. Which nursing intervention A. Heart and liver
is priority at this time? B. Heart and stomach
C. Liver and gallbladder
A. Call the operating room and schedule the D. Lungs and stomach
client for repeat surgery.
B. Notify the physician immediately 319. To limit the clients oxygen needs,a client with COPD
C. Check the colostomy for obstruction should be?
D. Explain to the client that this is expected in
A. Served three large meals a day
the next 2-3 days
B. Given six small meal a day
315. Which of the following characteristic of pain indicates C. Forced to drink large amount of fluid
gallbladder disease? D. Instructed to immediately sleep after meal

A. Left epigastria pain with cullens sing 320. The nurse is providing care to a client with a
B. Right lower quadrant pain with bloody tracheostomy tube in place. The nurse notes that his/her
diarrhea breathing has become noisy. Which intervention in a
C. Right upper quadrant pain that radiate to the nursing priority?
shoulder blade
A. Adjust the cuff pressure
D. Lower sterna pain that resembles indigestion.
B. Reposition the client
SITUATION; Mr jose dela cruz, age 68mis admitted to the C. Suction the tracheostomy
hospital due to fever, cough and dyspnea. His medical D. Notify the physician
history reveals long standing COPD.
SITUATION; Several clients are undergoing diagnostic
316. The caregiver of a client with COPD ask the nurse procedures.
Why cant we just increase the clients oxygen to help her
321. A client scheduled for electroencephalogram (EEG) is
breath easily the appropriate response of the nurse is?
being served breakfast. On her tray are the following:
A. let me notify the physician first scrambled egg, fried rice, dried fish and cup of coffee. What
B. Increasing oxygen levels may decrease the should the nurse do?
clients urge to breath
A. Withhold the meal

Fourth Year- BSN Page 32


500 Oncolgy Nursing Questionnaires

B. Remove the fried rice D. Identify the research problem and ask the
C. Remove the dried fish research questions
D. Remove the cup od coffee
327. If the researcher would like to establish the
322. The use of the snellens chart help to evaluate cranial relationship between the respondents educational status
nerve: and compliance to TB treatment, the dependent variable
is?
A. VI
B. IV A. TB clients
C. III B. Compliance to treatment
D. II C. Educational status
D. TB treatment
323. A client has a Glasgow coma scale score of 4. This is
interpreted to mean that the client is? 328. There are 300 TB clients enrolled for DOTS in the
municipality. What is the best sampling method to use?
A. Awake
B. Lethargic A. Simple random
C. Oriented B. Network
D. Unresponsive C. Purposive
D. Cluster
324. Which of the following diagnostic test does NOT
require informed consent? 329. If another researcher would like to do the same study,
which of the following is correct?
A. Colonoscopy
B. Electromyelography A. He/she should do a meta analysis
C. Electro encephalography B. He/she should just barrow the review of
D. Liver biopsy literature part of the proposal to save time
C. He/she can replicate the study
325. Which client does NOT have any risk from undergoing D. It is not ethical to be copying somebodys
MRI? study

A. A 35 year-old with insulin pump 330. If the department of health (DOH) would like to do a
B. A 55 year-old with cardiac pacemaker similar study on a national scale, what would be the most
C. A 28 year-old who weighs 310lbs appropriate sampling strategy?
D. A 19 year-old with pierced ear
A. Probability
SITUATION; A major concern in the public health is the B. Simple
development of resistant strain of TB bacteria. Mila, a C. Cluster
public health nurse, will conduct a research on the D. Stratified
compliance of TB clients and will look into the reasons
why many clients drop out of the short-course SITUATION; Mr. Mike Delos Reyes, R.N., is assigned to the
chemotherapy. medical ward. Severeal clients are admitted for treatment
ofvarious conditions.
326. To meet the aim of the study- to identify the factors
that affect TB clients compliance to therapy- the 331. The hallmark sign for the diagnosis of diabetes
researcher should? mellitus (DM) is the presence of?

A. Determine the purpose of the study A. Ketonuria


B. Acknowledgement the limitations of the study B. Acetone odor of breath
C. Interview clients who have problem in C. Kussmauls breathing
complying D. Hyperglycemia

Fourth Year- BSN Page 33


500 Oncolgy Nursing Questionnaires

332. Following stapedectomy with insertion of middle car C. Assess the client sense of taste in the
prosthesis, which of the following instructions should NOT posterior tongue
be given to the client? D. Use a tongue blade to check for the gag reflex

A. Avoid flying in small planes 337. Which of the following intervention is the highest
B. Avoid deep see diving priority of the nurse when carrying for a client with
C. Do not blow your nose until it is healed myasthenia gravis?
D. You will immediately notice an improvement
A. Controlling the blood pressure
333. Which of the following methods as best to obtain the B. Preventing injury
accurate amount of residual urine? C. Maintaining an airway
D. Promoting bowel elimination
A. remove the catheter slowly to ensure
drainage of the urine near the exit of the 338. A client with myasthenia gravis was admitted due to
bladder muscle weakness and bradycardia. Which drug will the
B. Press the lower abdomen while withdrawing nurse anticipate the doctor to order?
the catheter
C. Quickly full the catheter so any residual urine A. Atropine sulfate
is excreted abruptly B. Neostigmine
D. Slowly move the catheter in and out of the C. Pyridostigmine
bladder D. Diazepam ( Valium)

334. Which of the following manifestation is most 339. Muscle weakness also occurs in client with multiple
definitive of pheochromocytoma? sclerosis. Following treatment with steroid, an assessment
finding in a client with multiple sclerosis that indicates a
A. Headache positive response to therapy is decreased:
B. Hypertension
C. Hyperglycemia A. Mood swing
D. Hypermetabolism B. Muscle rigidity
C. Pain in the lower extremities
335. Which of the following characteristic of pain indicates D. Visual problem
gall bladder disease?
340. Mask-like faces occurs in clients with myasthenia
A. Left epigastric pain with Cullens sign gravis nd Parkinson disease. which of the following
B. Right lower quadrant pain with bloody nursing diagnosis us the highest priority for a client with
diarrhea Parkinsons disease?
C. Right upper quadrant pain that radiates to the
shoulder blade A. Altered nutrition; Less than body requirement
D. Lower sterna pain that resembles indigestion B. Risk for injury
C. Risk for aspiration
SITUATION; Mrs. Maria Delos Angeles, 45 years- old was D. Altered bowel elimination
admitted due ti sudden weakness of the lower extremities.
The nurse is assessing the client. SITUATION; In the emergency department, thenurse is
obtaining the history of several clients lined up for
336. How should the nurse assess the trigeminal nerve/ admission.

A. Ask the client to say ah and assess the 341. While playing in the garden, a 10 year-old boy had a
swallowing ability foreign object stuck in his left eye. which is intervention is
B. Used a needle to assess for facial sensation appropriate?

A. immediately irrigate the eye with warm water

Fourth Year- BSN Page 34


500 Oncolgy Nursing Questionnaires

B. Cover the eye loosely with sterile gauze and a. Cevix


refer to the doctor b. Stomach
C. Try to remove the object by blowing into the c. Skin melanomas
eye d. Bladder
D. Wait for the physician before touching the eye
348.Cancer death are most likely to occur in which group?
342. Which of the following nursing diagnosis is of the
highest priority for a client with Addisons disease? a. Children and adults
b. Young adults
A. Fluid and electrolyte imbalance c. Middle adults
B. Infection d. Older adults
C. Fluid volume excess
D. Altered cardiac output 349.Which of the following statements applies to all of the
present theories about the etiology of cancer?
343. The client has been receiving one liter of 5% dextrose
in normal saline. The total amount of this solution a. Cancer occurs only in young people
provides how many calories? b. There is impairment of the immune system
with cancer
A. 100 c. In cancer, proteins inhibit the growth of cells
B. 200 d. In cancer, the actions of oncogenes are
C. 350 repressed
D. 370
350.Mare Larrabee has cervical cancer. She asks the nurse
344. Which nursing diagnosis is of highest priority for a if it is possible that the cancer was caused by a virus. The
client with osteoarthritis? correct answer by the nurse is:

A. Impaired verbal communication a. Probably not, that is only one many theories.
B. ineffective breathing pattern b. All cancers have a viral etiology
C. Risk for injury c. I am not sure- you will have to ask your
D. Impaired nutrition: less than body doctor.
requirement d. Some cancers such as yours have a strong
association of viruses.
345. Which question is most appropriate to ask a client to
obtain information about his/her chest pain? 351.In which of the following cancer is heredity not
recognized as a risk factor?
A. Do you feel some pain
B. Do you feel some throbbing a. Breast cancer
C. Tell me about your pain b. Colon cancer
D. Were you doing something when the pain is c. Malignant melanoma
started d. Brain cancer

346. Cancer occurs when: 352.Which of the following clients probably has the least
occupational risk of developing cancer?
a. Normal cells multiple abnormally
b. Normal cells mutate into abnormal cells a. A coal miner
c. Abnormal cells become tumor cells b. A carpenter
d. Abnormal cells are exposed to toxins c. An oncology nurse
d. An English teacher
347. During the past 20 years which type of cancer has had
an increase in mortality rate? 353.Which of the following is characteristic of a benign
neoplasm?

Fourth Year- BSN Page 35


500 Oncolgy Nursing Questionnaires

a. Rapid growth a. A cancer


b. Invasive b. Metastatic disease
c. Noncohesive c. A malignancy
d. Well-defined borders d. A benign tumor

354.Tina Blackey was recently diagnosed with breast 359.In the TNM classification system, the N stands for:
cancer. She ask the nurse what steps she can take to boost
her immune system. The response by the nurse is: a. No metastasis
b. Node involvement
a. Refuse chemotherapy; it weakness the c. Numerical grade
immune system d. Name
b. Learn techniques to relieve or manage stress
more effectively 360.Which of the following explanations should the nurse
c. Ask your doctor to prescribe an utilize regarding the magnetic resonance imaging test used
antidepressant to diagnose a suspected brain tumor?
d. Research herbal remedies
a. This test uses sound waves
355.Joan Shipleys breast cancer has metastasized to b. Before this test, , the physician will inject you
lymph nodes. Which statement is correct regarding this with a radiopaque dye and then films will be
condition? taken
c. The machine for this test is like a tube, and
a. Metastasis means that Ms. Shipley will you will lie inside. It may make thumping
eventually die from cancer noises
b. The cancer has spread only to the lymph d. You must drink a radioactive substance
nodes before this test
c. Chemotherapy and radiation will be only
palliative in this situation
d. Aggressive treatment may reduce risks of 361.A client sys, The doctor explained so many things to
further spread of the cancer me that I forgot what she said about the endoscopy test Im
having tomorrow. The nurse should explain that in this
356.Which lab test is it most important to monitor for a test there is a direct visualization of the:
client with lung cancer that has ectopic sites?
a. Sigmoid colon
a. T3 b. Upper gastrointestinal tract
b. T4 c. Urethra
c. Serum glucose d. Tracheobronchial tree
d. D. blood culture
362.Jean Lawrence is undergoing diagnostic testing as a
357.Which statement about cancer is false? part of the staging process. She tells the nurse, I am so
worried they will find cancer everywhere: what will
a. Chronic cancer pain maybe treated-related happen if I die before my children are grow up? The most
b. Fear and hopelessness maybe contribute to appropriate response by the nurse is:
suffering of the client with cancer
c. Chronic cancer pain may affect objective a. There is no way of knowing what the future
manifestations holds.
d. Pain relief is not a goal of cancer care b. No matter what happens, your children will
be fine.
358.The MD has told a client that the results of the biopsy c. You sound like you are afraid.
reveal a papilloma. This statement means that the client d. Dont worry until you know the results.
has:

Fourth Year- BSN Page 36


500 Oncolgy Nursing Questionnaires

363.Which of the following client statements indicate the 368.When preparing and administering chemotherapy and
need for further client-teaching about chemotherapy? disposing of the equipment, the nurse should:

a. Chemotherapy is local treatment. a. Use universal precautions


b. I may experience nausea, vomiting and hair b. Wear gloves
loss during the treatments. c. Wear gloves, a mask and a gown
c. I may receive both oral and IV medications. d. Wear gloves and mask
d. I may receive a combination of drugs over
different periods of time. 369.Marlene Marrison is scheduled to start chemotherapy.
Which statement by Mrs. Marrison indicates the need for
364.Which of the following are is true about the cell-kill further client-teaching?
hypothesis?
a. I will have to drink plenty of fluids each day.
a. It explains the bodys mechanism of b. Visits from my school-aged grandchildren
protection fro carcinogens will help my spirits.
b. It forms the basis of selecting specific classes c. I will need extra rest during this period of
of chemotherapeutic drugs time.
c. It accounts the severity of side effects. d. I may need a extra help at home to clean and
d. It is the reason why several courses of prepare meals.
chemotherapy are given.
370.Shelley Nordant is scheduled for surgery remove a
365.An example of a chemotherapy catheter that is cancerous tumor. She remarks to the nurse, Why have
tunneled under the skin on the chest into the subclavian surgery? I am going to have a chemotherapy just the
vein is: same. The most appropriate response by the nurse is to:

a. Hickman a. Tell Mrs. Nordant that surgery must be


b. Peripherally inserted central catheter (PICC) performed.
c. Mediport b. Encourage her to confer with the surgeon and
d. Port-a-cath the oncologist about the surgery
c. Tell her that the combination of surgery and
366.Which of the following nursing interventions is radiation may lead to cure
contraindicated for a client receiving Cytoxan d. Document her concerns and give her
(cycloplasphamide) reassurance

a. Monitor clients WBC, BUN, and liver enzyme


lab data
b. Teach client about ways to manage hair loss 371. Jeovina, with advanced breast cancer is prescribed
c. Restrict daily fluid intake to 1L during tamoxifen (Nolvadex). When teaching the client about this
treatment drug, the nurse should emphasize the importance of
d. Encourage the client to use relaxation reporting which adverse reaction immediately?
techniques a. Vision changes
b. Hearing loss
367.Assessing for arrythmiasis, gallops, and congestive c. Headache
heart failure is an especially important nursing d. Anorexia
intervention for a client receiving:
372. A female client with cancer is being evaluated for
a. Vinblastin (Velban) possible metastasis. Which of the following is one of the
most common metastasis sites for cancer cells?
b. Doxuribican (Adriamycin)
a. Liver
c. 5-Fluorouracil (5-FU) b. Colon
d. Vincristine (Onvocin)

Fourth Year- BSN Page 37


500 Oncolgy Nursing Questionnaires

c. Reproductive tract 377. The nurse is preparing for a female client for
d. White blood cells (WBCs) magnetic resonance imaging (MRI) to confirm or rule out a
spinal cord lesion. During the MRI scan, which of the
373. A 34-year-old female client is requesting information following would pose a threat to the client?
about mammograms and breast cancer. She isnt a. The client lies still.
considered at high risk for breast cancer. What should the b. The client asks questions.
nurse tell this client? c. The client hears thumping sounds.
a. She should have had a baseline mammogram before d. The client wears a watch and wedding band.
age 30.
b. She should eat a low-fat diet to further decrease her 378.. Nina, an oncology nurse educator is speaking to a
risk of breast cancer. womens group about breast cancer. Questions and
c. She should perform breast self-examination during the comments from the audience reveal a misunderstanding of
first 5 days of each menstrual cycle. some aspects of the disease. Various members of the
d. When she begins having yearly mammograms, breast audience have made all of the following statements. Which
self-examinations will no longer be necessary. one is accurate?
a. Mammography is the most reliable method for
374. Nurse Brian is developing a plan of care for marrow detecting breast cancer.
suppression, the major dose-limiting adverse reaction to b. Breast cancer is the leading killer of women of
floxuridine (FUDR). How long after drug administration childbearing age.
does bone marrow suppression become noticeable? c. Breast cancer requires a mastectomy.
a. 24 hours d. Men can develop breast cancer.
b. 2 to 4 days
c. 7 to 14 days 379.. Nurse Meredith is instructing a premenopausal
d. 21 to 28 days woman about breast self-examination. The nurse should
tell the client to do her self-examination:
Mr. Danny Ang, an alcoholic, is admitted to the a. at the end of her menstrual cycle.
hospital due to cirrhosis of the liver. b. on the same day each month.
c. on the 1st day of the menstrual cycle.
d. immediately after her menstrual period.
375.. Which of the following is the early symptom that
he may manifest and would lead to progression 380.. Nurse Kent is teaching a male client to perform
into hepatic coma? monthly testicular self-examinations. Which of the
A. Polyuria following points would be appropriate to make?
C. Nystagmus a. Testicular cancer is a highly curable type of cancer.
B. Dysuria b. Testicular cancer is very difficult to diagnose.
D. Behavioral changes c. Testicular cancer is the number one cause of cancer
deaths in males.
d. Testicular cancer is more common in older men.
376. What would include the symptoms indicating that
three is progression into hepatic coma? 381. Rhea, has malignant lymphoma. As part of her
1. Nystagmus chemotherapy, the physician prescribes chlorambucil
2. Fetid breath (Leukeran), 10 mg by mouth daily. When caring for the
3. Flapping tremor client, the nurse teaches her about adverse reactions to
4. Fruity tremor chlorambucil, such as alopecia. How soon after the first
5. Fruity odor breath administration of chlorambucil might this reaction occur?
a. Immediately
A. 2 and 4
b. 1 week
C. 2 and 3 c. 2 to 3 weeks
B. 1 and 2 d. 1 month
D. 2 and 3
382. What is rationale of giving him low-protein diet?
A. Decrease formation of ammonia
B. Decrease production of bite
C. Increase formation of ammonia
Fourth Year- BSN Page 38
500 Oncolgy Nursing Questionnaires

D. Decrease the desire for alcohol B. Encourage the family members to raise
the issue of dying
383.. Because of ineffective breathing pattern, how C. Verify what the physician has told Mrs. M
would you improve his respiratory status: about her diagnosis and prognosis before
A. Conserve his strength initiating a discussion
B. Maintain the desired position D. Remain available and listen carefully for
C. Elevate foot of bed cues Mr. M is ready to discuss her
D. Assist him after the paracentesis impending death

388. Mrs. M begins to deteriorate markedly. She


requires frequent pain medication and her oldest
384.. A male client is receiving the cell cyclenonspecific daughter, Izza age 20, walks out of the room and crying
alkylating agent thiotepa (Thioplex), 60 mg weekly for 4 and says to the nurse, I can not take this anymore. What
weeks by bladder instillation as part of a would be the most appropriate response of the nurse?
chemotherapeutic regimen to treat bladder cancer. The
A. I hope your mother didnt hear you. She
client asks the nurse how the drug works. How does
thiotepa exert its therapeutic effects? needs your support right now.
a. It interferes with deoxyribonucleic acid (DNA) B. It is really difficult to see someone you
replication only. love suffer. Let us go outside and talk
b. It interferes with ribonucleic acid (RNA) transcription about how you are feeling.
only. C. You say that you dont take it anymore.
c. It interferes with DNA replication and RNA You should also considered what she is
transcription.
feeling.
d. It destroys the cell membrane, causing lysis.
D. She will probably die soon. She is getting
385.. The nurse is instructing the 35 year old client to weaker everyday.
perform a testicular self-examination. The nurse tells the 389.. The nurse performs one of there roles when she
client: listens to Mrs. M and his family:
a. To examine the testicles while lying down A. Parent surrogate
b. That the best time for the examination is after a C. Socializing agent
shower B. Teacher
c. To gently feel the testicle with one finger to feel for a
D. Counselor
growth
d. That testicular self-examination should be done at
least every 6 months

386. A female client with cancer is receiving 390. Gian, a community health nurse is instructing a
chemotherapy and develops thrombocytopenia. The nurse group of female clients about breast self-examination. The
identifies which intervention as the highest priority in the nurse instructs the client to perform the examination:
nursing plan of care? a. At the onset of menstruation
a. Monitoring temperature b. Every month during ovulation
b. Ambulation three times daily c. Weekly at the same time of day
c. Monitoring the platelet count d. 1 week after menstruation begins
d. Monitoring for pathological fractures
391.. Nurse Cecilia is caring for a client who has
387. The nurse knows that a dying patient should be undergone a vaginal hysterectomy. The nurse avoids
encouraged to talk about death and dying. The which of the following in the care of this client?
therapeutic approach is to: a. Elevating the knee gatch on the bed
b. Assisting with range-of-motion leg exercises
A. Confront Mrs. M with the facts about her
c. Removal of antiembolism stockings twice daily
illness and impending death to open d. Checking placement of pneumatic compression boots
discussion

Fourth Year- BSN Page 39


500 Oncolgy Nursing Questionnaires

392.. Mina, who is suspected of an ovarian tumor is B. Involves bladder, rectum and outside
scheduled for a pelvic ultrasound. The nurse provides pelvic
which preprocedure instruction to the client? C. Extends outside corpus but not outside
a. Eat a light breakfast only
pelvic
b. Maintain an NPO status before the procedure
c. Wear comfortable clothing and shoes for the procedure D. Is confined to corpus only
d. Drink six to eight glasses of water without voiding
before the test

393.. A male client is diagnosed as having a bowel tumor


and several diagnostic tests are prescribed. The nurse 397.. A female client diagnosed with multiple myeloma
understands that which test will confirm the diagnosis of and the client asks the nurse about the diagnosis. The
malignancy? nurse bases the response on which description of this
a. Biopsy of the tumor disorder?
b. Abdominal ultrasound a. Altered red blood cell production
c. Magnetic resonance imaging b. Altered production of lymph nodes
d. Computerized tomography scan c. Malignant exacerbation in the number of leukocytes
d. Malignant proliferation of plasma cells within the bone
394.Trisha, a 56-years old executive consults her
gynecologist due to vaginal bleeding. Diagnostic D 398.. Nurse Bea is reviewing the laboratory results of a
& C reveals uterine cancer, Stage II. She is client diagnosed with multiple myeloma. Which of the
scheduled for total abdominal hysterectomy with following would the nurse expect to note specifically in
this disorder?
bilateral salpingoophorectomy.
a. Increased calcium
b. Increased white blood cells
. Trisha expresses her fear about the surgery. c. Decreased blood urea nitrogen level
Which of the following statements is the beast d. Decreased number of plasma cells in the bone marrow
response of the nurse?
A. Let us focus on your post-operative care 399.. Vanessa, a community health nurse conducts a
B. Your gynecologist is very competent. You health promotion program regarding testicular cancer to
are surely in safe hands community members. The nurse determines that further
information needs to be provided if a community member
C. Tell me about your fears. I am here to
states that which of the following is a sign of testicular
listen cancer?
D. Do not be afraid because it may affect a. Alopecia
your recovery b. Back pain
c. Painless testicular swelling
395. Which of the following is a priority nursing d. Heavy sensation in the scrotum
diagnosis immediately after the surgery?
A. Disturbance in body image related to loss 400 The male client is receiving external radiation to the
neck for cancer of the larynx. The most likely side effect to
of uterus
be expected is:
B. Fluid volume deficit to surgery a. Dyspnea
C. Abdominal pain related to b. Diarrhea
surgical incision c. Sore throat
D. Altered breathing pattern related to d. Constipation
anesthesia
401. Nurse Joy is caring for a client with an internal
396.. Three days post surgery, the nurse observe that radiation implant. When caring for the client, the nurse
should observe which of the following principles?
Trisha understands her disease condition when
a. Limit the time with the client to 1 hour per shift
she remarks that Stage II uterine cancer. b. Do not allow pregnant women into the clients room
A. Involves corpus and cervix c. Remove the dosimeter badge when entering the
clients room

Fourth Year- BSN Page 40


500 Oncolgy Nursing Questionnaires

d. Individuals younger than 16 years old may be allowed a. I will observed my husband for any
to go in the room as long as they are 6 feet away from the chances of mental status.
client
b. I understand how to give the drug
402. Which of the following statements best subcutaneously.
describes therapy? c. I can have serve flulike symptoms while
a. Bradytherapy involves placing radioactive taking these drugs.
materials at some distance from the tumor d. I will limit my fluid intake.
b. Bradytherapy involves placing radioactive 406. A nurse using the Karnofsky scale to assess a
materials directly into tumor site client with cancer is evaluating the client:
c. Bradytherapy is a holistic, nontraditional a. Amount of pain
cancer treatment b. Stress level
d. Bradytherapy is also referred to teletherapy c. Performance status
d. Nutritional intake
403. Which of the following nursing actions is most 407. Which finding would support a diagnosis of
appropriate when caring for client with unsealed poor nutrition/
implanted radioisotope? a. Oral mucous membranes pink-red and
a. Wear an abdominal apron moist
b. Have the other client in the room wear b. Tongue bright to dark red and swollen
monitoring device to measure exposure c. Slight hair loss
c. Pick up any dislodged implants found in bed d. Acne
linen with your sterile gloved hand 408. Stuart Eggleton, a client with prostate cancer, is
d. Dispose of clients body fluid in specially trembling and is pacing his room. His blood
marked containers pressure and pulse are slightly elevated. Mr.
Eggleton may be experiencing:
a. Some type of drug reaction
b. Anxiety
404. Which of the following statements indicates
c. Body image disturbance
that the client-teaching prior to an external
d. Fear of the unknown
radiation was successful?
409. Which of the following statements indicates
a. If I have pain at my treatment site, I will
that the client has misunderstood client-teaching
apply an ice pack.
about alopecia from chemotherapy?
b. I will not have any intimate sexual contact
a. I can wear a colorful scarf of turban.
while Im receiving treatment.
b. I have already bought a wig.
c. I should protect my skin from sun exposure
c. My neighbor will be driving me to the
during treatments.
Look Good, Feel Better program.
d. I will wash the skin in the treatment area
d. After the chemotherapy is finished, my
daily using soap and hot water
hair will grow back as the same it was
405. George Houng is receiving combination therapy
before the treatment.
of alpha-interferon with another chemotherapeutic
410. Which finding indicates a possible infection in
agent. Which statement indicates that the client
the client with cancer who is immunosuppressed?
and family require further teaching from the nurse/
a. A decreased respiratory rate
b. A decreased pulse rate

Fourth Year- BSN Page 41


500 Oncolgy Nursing Questionnaires

c. A normal temperature c. Prevent dry mouth by using moisturizing


d. An increased blood pressure agents such as Blistex
411. The partner of a client with cancer has been d. Teach client to use smooth rather than
provided with guidelines regarding when to call for sharp utensils for eating.
help. Which of the following statements by the 415. Irina Landrigan is 49 year-old client who has just
clients partner indicates further teaching about the undergone a mastectomy. When the nurse is
guidelines is needed? performing the first dressing change, Mrs.
a. Ill contact the physician if I see ant new Landrigan states, I just do not want to look at it.
bleeding from her rectum. The most appropriate response by the nurse is:
b. If her oral temperature is 100F, Ill notify a. You are going to have to look at it some
the physician. time.
c. If she becomes withdrawn and cries b. You do not have to look at your incision.
frequently, Ill notify the physician. c. You will eventually be able to look at it.
d. Ill contact the physician If I notice a d. Every woman with this kind of surgery
change in her eating patterns. feels that way.
412. Which f the following nursing assessment would 416. The doctor has told Javier Rodriguez and his
indicate an improvement in a cancer client with family that has about 6 to 9 months to live. Yhe
nursing diagnosis Altered Nutrition: Less than the family asks the nurse about making funeral
body Requirements? arrangements ahead of time. The most appropriate
a. Serum albumin less than 3.0 response by the nurse is:
b. A PPD response of more than 5 mm of a. Many people do make arrangements
induration ahead, but you might upset Mr. Rodriguez if
c. A decrease in urinary creatinine level you tell him about the plans.
d. An anthropometric measurement of 75% of b. If you would like to do this, you should try
the standard to include Mr. Rodriguez in the planning.
413. Which of the following nursing intervention c. Dont give up hope; he may still respond to
would be most helpful in managing nausea and therapy.
vomiting of a client with cancer? d. You should wait until Mr. Rodriguez dies
a. Instruct the client to drink plenty of fluids before making funeral plans.
each meal 417. Which strategy would not be recommended to
b. Instruct the client to lie down for an hour of a cancer client with problems eating?
eating a. Eat icy-cold foods
c. Encourage the client to eat dry foods b. Eat frequent, small meals
d. Instruct the client to follow high-fat food c. Eat semisoft and liquid foods
plan d. Eat spicy, warm foods
414. Which of the following nursing actions is 418. Which of the following statements indicates
contraindicated for the client with leukemia? that the teaching about the American Cancer
a. Assess the clients oral mucous membranes Society s dietary guidelines to prevent was
daily understood by the team?
b. Instruct the client to brush and floss the a. I should take vitamin supplements ever
teeth three ties per day day.

Fourth Year- BSN Page 42


500 Oncolgy Nursing Questionnaires

b. My food pattern should include serving of down the toilet


salt-cured foods four times a week. d. Pick up the implant with long-handled forceps and
place it in a lead container.
c. I will include high-fat foods in my meal
plan. 423. The nurse is caring for a female client experiencing
d. eating more high-fiber foods will be better neutropenia as a result of chemotherapy and develops a
plan of care for the client. The nurse plans to:
for me.
a. Restrict all visitors
419. Dana Biencarz has a lung cancer and is b. Restrict fluid intake
experiencing facial and arm edema. The nurse c. Teach the client and family about the need for hand
should suspected that Mrs. Biancarz has: hygiene
d. Insert an indwelling urinary catheter to prevent skin
a. Allergic reaction breakdown
b. Pericardial effusion
c. Sepsis 424. The home health care nurse is caring for a male
client with cancer and the client is complaining of acute
d. Superior vena cava syndrome
pain. The appropriate nursing assessment of the clients
420. Rita Lopez has metastatic breast cancer and is pain would include which of the following?
admitted to the hospital for pain management. a. The clients pain rating
b. Nonverbal cues from the client
During the night she becomes incontinent, which
c. The nurses impression of the clients pain
has never occurred before. The appropriate action d. Pain relief after appropriate nursing intervention
by the nurse is to:
a. Recognize that the incontinence occurred 425.. The nurse is interviewing a male client about his
past medical history. Which preexisting condition may
as a result of oversedation
lead the nurse to suspect that a client has colorectal
b. Notify the physician that the spinal cord cancer?
compression is suspected a. Duodenal ulcers
b. Hemorrhoids
c. Realize that incontinence can occur in
c. Weight gain
metastatic bone disease d. Polyps
d. Wait until the morning to see if the
incontinence recurs 426.. Nurse Amy is speaking to a group of women about
early detection of breast cancer. The average age of the
women in the group is 47. Following the American Cancer
Society guidelines, the nurse should recommend that the
women:
a. perform breast self-examination annually.
b. have a mammogram annually.
421.. A cervical radiation implant is placed in the client c. have a hormonal receptor assay annually.
for treatment of cervical cancer. The nurse initiates what d. have a physician conduct a clinical examination every
most appropriate activity order for this client? 2 years.
a. Bed rest
b. Out of bed ad lib 427. A male client with a nagging cough makes an
c. Out of bed in a chair only appointment to see the physician after reading that this
d. Ambulation to the bathroom only symptom is one of the seven warning signs of cancer. What
is another warning sign of cancer?
422.. A female client is hospitalized for insertion of an a. Persistent nausea
internal cervical radiation implant. While giving care, the b. Rash
nurse finds the radiation implant in the bed. The initial c. Indigestion
action by the nurse is to: d. Chronic ache or pain
a. Call the physician
b. Reinsert the implant into the vagina immediately 428.. For a female client newly diagnosed with radiation-
c. Pick up the implant with gloved hands and flush it induced thrombocytopenia, the nurse should include

Fourth Year- BSN Page 43


500 Oncolgy Nursing Questionnaires

which intervention in the plan of care? a. women at higher risk for benign proliferative breast
a. Administering aspirin if the temperature exceeds 102 disease.
F (38.8 C) b. women at low risk for breast cancer.
b. Inspecting the skin for petechiae once every shift c. screening women over age 65.
c. Providing for frequent rest periods d. women with breast implants.
d. Placing the client in strict isolation

429.. Nurse Lucia is providing breast cancer education at


a community facility. The American Cancer Society 434. The nurse recognizes which of the following
recommends that women get mammograms: statements as accurately reflecting a risk factor for breast
a. yearly after age 40. cancer?
b. after the birth of the first child and every 2 years
thereafter.
a. Mother affected by cancer before 60 years of age
c. after the first menstrual period and annually
b. Onset of menses before 14 years of age
thereafter.
c. Multiparity
d. every 3 years between ages 20 and 40 and annually
d. No alcohol consumption
thereafter.

435. The nurse is teaching a client about the risk factors


430. A male client is in isolation after receiving an
associated with colorectal cancer. The nurse determines
internal radioactive implant to treat cancer. Two hours that further teaching related to colorectal cancer is
later, the nurse discovers the implant in the bed linens. necessary if the client identifies which of the following as
What should the nurse do first? an associated risk factor?
a. Stand as far away from the implant as possible and call
for help. a. Age younger than 50 years
b. Pick up the implant with long-handled forceps and b. History of colorectal polyps
place it in a lead-lined container. c. Family history of colorectal cancer
d. Chronic inflammatory bowel disease
c. Leave the room and notify the radiation therapy
department immediately.
435. Answer: A
d. Put the implant back in place, using forceps and a
Rationale: Colorectal cancer risk factors include age older
shield for self-protection, and call for help. than 50 years, a family history of the disease, colorectal
polyps, and chronic inflammatory bowel disease.

436. When teaching a client about the signs of colorectal


431. The nurse is admitting a client with laryngeal cancer cancer, Nurse Trish stresses that the most common
to the Nursing unit. The nurse assesses for which most complaint of persons with colorectal cancer is:
common risk factor for this type of cancer?
a. Abdominal Pain
a. Alcohol abuse b. Hemorrhoids
b. Cigarette smoking c. Change in caliber of stools
c. Use of chewing tobacco d. Change in bowel habits
d. Exposure to air pollutants
436. Answer: D
432. The nurse recognizes which of the following Rationale: Constipation, diarrhea, and/or constipation
statements as accurately reflecting a risk factor for breast alternating with diarrhea are the most common symptoms
cancer? of colorectal cancer

a. Mother affected by cancer before 60 years of age


b. Onset of menses before 14 years of age
c. Multiparity 438. The nurse is teaching a client about the risk factors
d. No alcohol consumption associated with colorectal cancer. The nurse determines
that further teaching related to colorectal cancer is
433. Ductal lavage is used for

Fourth Year- BSN Page 44


500 Oncolgy Nursing Questionnaires

necessary if the client identifies which of the following as


an associated risk factor?
441.The nurse is caring for a client diagnosed with
a. Age younger than 50 years squamous cell skin cancer and writes a psychosocial
b. History of colorectal polyps problem of fear. Which Nursing interventions should be
c. Family history of colorectal cancer included in the plan of care?
d. Chronic inflammatory bowel disease
a. Explain to the client that the fears are unfounded.
438. Answer: A b. Encourage the client to verbalize the feeling of being
Rationale: Colorectal cancer risk factors include age older afraid.
than 50 years, a family history of the disease, colorectal c. Have the HCP discuss the clients fear with the client.
polyps, and chronic inflammatory bowel disease. d. Instruct the client regarding all planned procedures.

441.Answer: B
Rationale: This is the most commonly written therapeutic
439. The 85-year-old male client diagnosed with cancer communication goal. This addresses the clients concerns.
of the colon asks the nurse, Why did I get this cancer?
Which statement is the nurses best response?

a. Cancer of the colon is associated with a lack of fiber in 442. The 33-year-old client diagnosed with Stage IV
the diet. Hodgkins lymphoma is at the five (5)- year remission
b. Cancer of the colon has a greater incidence among those mark. Which information should the nurse teach the
younger than age 50 years. client?
c. Cancer of the colon has no known risk factors.
d. Cancer of the colon is rare among male clients. a. Instruct the client to continue scheduled screenings for
cancer.
439. Answer: A b. Discuss the need for follow-up appointments every five
Rationale: A long history of low-fiber, high-fat, high (5) years.
protein diets results in a prolonged transit time. This c. Teach the client that the cancer risk is now the same as
allows the carcinogenic agents in the waste products to for the general population.
have a greater exposure to the lumen of the colon. d. Have the client talk with the family about funeral
arrangements.

442. Answer: A
440. The nurse has been giving instructions to Mrs. S., Rationale: The five (5)-year mark is a time for celebration
a c5-year-old white female, about preventing skin for clients diagnosed with cancer, but the therapies can
cancer. Which statement best indicates Mrs. S.s cause secondary malignancies and there may be a genetic
understanding of skin cancer risk factors? predisposition for the client to develop cancer. The client
should continue to be tested regularly.
a. My father was treated for melanoma, but my mom says
not to worry.
b. I really need to use sunscreeneven in winter.
c. I guess because I am dark complected I will be more 443.Which clinical manifestation of Stage I non-
prone to developing skin cancer. Hodgkins lymphoma would the nurse expect to find
d. I used to lay in the sun all the timenow I just go to the when assessing the client?
tanning bed.
a. Enlarged lymph tissue anywhere in the body.
440. Correct Answer: B b. Tender left upper quadrant.
Rationale: Almost all cases of basal and squamous cell skin c. No symptom in this stage.
cancer diagnosed each year in the United States are d. Elevated B cell lymphocytes on the CBC.
considered to be sun-related.

Fourth Year- BSN Page 45


500 Oncolgy Nursing Questionnaires

444. The nurse is admitting a client with rule-out 449. The occupational health nurse is preparing a
Hodgkins lymphoma. When the nurse assesses the client, presentation to a group of factory workers about
which data would support this diagnosis? preventing colon cancer. Which information should be
included in the presentation?
a. Night sweats and fever without chills.
b. Edematous lymph nodes in the groin. a. Wear a high filtration mask when around chemicals.
c. Malaise and complaints of an upset stomach. b. Eat several servings of cruciferous vegetables daily.
d. Pain in the neck area after a fatty meal. c. Take a multiple vitamin every day.
d. Do not engage in high-risk sexual behaviors.

450. A client is being evaluated for cancer of the colon. In


445. A client admitted with newly diagnosed with preparing the client for barium enema, the nurse should:
Hodgkins disease. Which of the following would the
nurse expect the client to report? a. Give laxative the night before and a cleansing enema in
the morning before the test
a. Lymph node Pain b. Render an oil retention enema and give laxative the
b. Weight gain night before
c. Night sweats c. Instruct the client to swallow 6 radiopaque tablets the
d. Headache evening before the study
d. Place the client on CBR a day before the study
446. he client is diagnosed with Cancer of the head of the
Pancreas. When assessing the patient, which signs and 451.The nurse is caring for the client scheduled for an
symptoms would the nurse expect to find? abdominal perineal resection for Stage IV colon cancer.
When preparing the plan of care during surgery, which
client problem should the nurse include in the plan?
a. Clay-colored stools and dark urine.
b. Night sweats and fever.
c. Left lower abdominal cramps and tenesmus. a. Fluid volume deficit.
d. Nausea and coffee-ground emesis. b. Impaired tissue perfusion.
c. Infection of surgical site.
d. Immunosuppression.
447. The nurse is planning a program for clients at a health
fair regarding the prevention and early Detection of
Cancer of the Pancreas. Which self-care activity should
the nurse teach that is an example of primary Nursing
care? 452. The nurse is reviewing the preoperative orders of a
client with a colon tumor who is scheduled for abdominal
a. Monitor for elevated blood glucose at random intervals. perineal resection and notes that the physician has
b. Inspect the skin and sclera of the eyes for a yellow tint. prescribed neomycin (Mycifradin) for the client. The nurse
c. Limit meat in the diet and eat a diet that is low in fats. determines that this medication has been prescribed
d. Instruct the client with hyperglycemia about insulin primarily:
injections.
a. To prevent an immune dysfunction
448. The community nurse is conducting a health b. Because the client has an Infection
promotion program and the topic of the discussion relates c. To decrease the bacteria in the bowel
to the risk factors for gastric cancer. d. Because the client is allergic to penicillin
Which risk factor, if identified by a client, indicates a need
for further discussion? The nurse is admitting a male client to a medical floor with
a diagnosis of adenocarcinoma of the rectosigmoid
a. Smoking colon. Which assessment data support this diagnosis?
b. A high-fat diet
c. Foods containing nitrates a. The client reports up to 20 bloody stools per day.
d. A diet of smoked, highly salted, and spiced food b. The client states that he has a feeling of fullness after a
heavy meal.
c. The client has diarrhea alternating with constipation.

Fourth Year- BSN Page 46


500 Oncolgy Nursing Questionnaires

d. The client complains of right lower quadrant Pain with a. Sleep with the head of the bed elevated to prevent
rebound tenderness. increased intracranial pressure.
b. Take an analgesic medication for Pain only when the
454. During the admission assessment of a client with Pain becomes severe.
advanced ovarian cancer, the nurse recognizes which c. Explain that radiation therapy to the head may result in
symptom as typical of the disease? permanent hair loss.
d. Discuss end-of-life decisions prior to cognitive
a. Diarrhea deterioration.
b. Hypermenorrhea
c. Abnormal bleeding
d. Abdominal distention
459. A client with leukemia is undergoing radiation
455. The client with ovarian cancer is being treated with therapy to the brain and spinal cord. In planning care for
vincristine (Oncovin, Vincasar PFS). The nurse monitors this client, the nurse would include which Nursing
the client, knowing that which of the following indicates a intervention?
side effect specific to this medication?
a. A scalp ointment to prevent dryness
a. Diarrhea b. Avoiding washing off the targets marks
b. Hair loss c. Not allowing the client to use a hat or scarf
c. Chest Pain d. A dandruff shampoo twice daily
d. Numbness and tingling in the fingers and toes
Marie with acute lymphocytic leukemia suffers from
nausea and headache. These clinical manifestations may
indicate all of the following except
456. The client with small cell lung cancer is being
treated with etoposide (VePesid). The nurse monitors the a. effects of radiation
client during administration, knowing that which of the b. chemotherapy side effects
following indicates a side effect specific to this medication? c. meningeal irritation
d. gastric distension
a. Alopecia
b. Chest Pain 461. The nurse has reviewed a discharge teaching checklist
c. Pulmonary fibrosis for a client, a 65-year-old male with chronic lymphocytic
d. Orthostatic hypotension leukemia (CLL). Which of the following statements by the
client would indicate to the nurse that further review is
necessary?

457. A female client with carcinoma of the breast is a. Im retired, so I can sleep whenever I want.
admitted to the hospital for treatment with intravenously b. Ive got season tickets for all the basketball games.
administered doxorubicin (Adriamycin). The client tells c. Ill call the doctor if I have fever greater than 99F.
the nurse that she has been told by herfriends that she is d. Im going to teach my grandson how to fish.
going to lose all her hair. The appropriate Nursing
response is which of the following? 462. The client is diagnosed with chronic lymphocytic
leukemia (CLL) after routine laboratory tests during a
a. Your friends are correct. yearly physical. Which is the scientific rationale for the
b. You will not lose your hair. random nature of discovering the illness?
c. Hair loss may occur, but it will grow back just as it is
now. a. CCL is not serious, and clients die from other causes first.
d. Hair loss may occur, and it will grow back, but it may b. There are no symptoms with this form of leukemia.
have a different color or texture. c. This is a childhood illness and is self-limiting.
d. In early stages of CLL the client may be asymptomatic.
458. The client diagnosed with leukemia has central
nervous system involvement. Which instructions should
the nurse teach?

Fourth Year- BSN Page 47


500 Oncolgy Nursing Questionnaires

463. The client diagnosed with leukemia is being about chemotherapy is true?
admitted for an induction course of chemotherapy. A. it is a local treatment affecting only tumor cells
Which laboratory values indicate a diagnosis of leukemia? B. it affects both normal and tumor cells
C. it has been proven as a complete cure for cancer
a. A left shift in the white blood cell count differential. D. it is often used as a palliative measure.
b. A large number of WBCs that decreases after the
administration of antibiotics.
c. An abnormally low hemoglobin (Hgb) and hematocrit
(Hct) level. 469. . Which is an incorrect statement pertaining to the
d. Red blood cells that are larger than normal. following procedures for cancer diagnostics?
A. Biopsy is the removal of suspicious tissue and the only
464. The nurse is assessing a client diagnosed with acute definitive method to diagnose cancer
myeloid leukemia. Which assessment data support this B. Ultrasonography detects tissue density changes difficult
diagnosis? to observe by X-ray via sound waves.
C. CT scanning uses magnetic fields and radio frequencies
a. Fever and infections.
to provide cross-sectional view of tumor
b. Nausea and vomiting.
c. Excessive energy and high platelet counts. D. Endoscopy provides direct view of a body cavity to
d. Cervical lymph node enlargement and positive acid-fast detect abnormality.
bacillus.
470.. A post-operative complication of mastectomy is
lymphedema. This can be prevented by

465. Which of the following observations reported by a A. ensuring patency of wound drainage tube
patient with acute myelogenous leukemia (AML) would B. placing the arm on the affected side in a dependent
the nurse first assess? position
C. restricting movement of the affected arm
a. Weakness and fatigue
D. frequently elevating the arm of the affected side above
b. Bruising on the arm
c. Drainage from a small finger cut the level of the heart.
d. Mild abdominal Pain
.471. Which statement by the client indicates to the nurse
466.. Mr. Perez is in continuous pain from cancer that has that the patient understands precautions necessary during
metastasized to the bone. Pain medication provides little internal radiation therapy for cancer of the cervix?
relief and he refuses to move. The nurse should plan to:
A. Reassure him that the nurses will not hurt him A. I should get out of bed and walk around in my room.
B. Let him perform his own activities of daily living B. My 7 year old twins should not come to visit me while
C. Handle him gently when assisting with required care Im receiving treatment.
D. Complete A.M. care quickly as possible when necessary C. I will try not to cough, because the force might make me
expel the application.
467. . A client, who is suspected of having D. I know that my primary nurse has to wear one of those
Pheochromocytoma, complains of sweating, palpitation badges like the people in the x-ray department, but they
and headache. Which assessment is essential for the nurse are not necessary for anyone else who comes in here.
to make first?
A. Pupil reaction \472.. The laboratory results of the client with leukemia
B. Hand grips indicate bone marrow depression. The nurse should
C. Blood pressure encourage the client to:
D. Blood glucose
A. Increase his activity level and ambulate frequently
468. . A chemotherapeutic agent 5FU is ordered as an B. Sleep with the head of his bed slightly elevated
adjunct measure to surgery. Which of the ff. statements C. Drink citrus juices frequently for nourishment

Fourth Year- BSN Page 48


500 Oncolgy Nursing Questionnaires

D. Use a soft toothbrush and electric razor A. Changing the abdominal dressing
B. Maintaining patency of the cystotomy tube
C. Maintaining patency of a three-way Foley catheter for
473. Dennis receives a blood transfusion and develops cystoclysis
flank pain, chills, fever and hematuria. The nurse D. Observing for hemorrhage and wound infection
recognizes that Dennis is probably experiencing:

A. An anaphylactic transfusion reaction 478. In the early postoperative period following a


B. An allergic transfusion reaction transurethral surgery, the most common complication the
C. A hemolytic transfusion reaction nurse should observe for is:
D. A pyrogenic transfusion reaction
A. Sepsis
B. Hemorrhage
474.. A client jokes about his leukemia even though he is C. Leakage around the catheter
becoming sicker and weaker. The nurses most therapeutic D. Urinary retention with overflow
response would be:

A. Your laugher is a cover for your fear. 479.. Following prostate surgery, the retention catheter is
B. He who laughs on the outside, cries on the inside. secured to the clients leg causing slight traction of the
C. Why are you always laughing? inflatable balloon against the prostatic fossa. This is done
D. Does it help you to joke about your illness? to:

475. n dealing with a dying client who is in the denial stage A. Limit discomfort
of grief, the best nursing approach is to: B. Provide hemostasis
C. Reduce bladder spasms
A. Agree with and encourage the clients denial D. Promote urinary drainage
B. Reassure the client that everything will be okay
C. Allow the denial but be available to discuss death 480.Twenty-four hours after TURP surgery, the client tells
D. Leave the client alone to discuss the loss the nurse he has lower abdominal discomfort. The nurse
notes that the catheter drainage has stopped. The nurses
476.. Rene, age 62, is scheduled for a TURP after being initial action should be to:
diagnosed with a Benign Prostatic Hyperplasia (BPH). As
part of the preoperative teaching, the nurse should tell the A. Irrigate the catheter with saline
client that after surgery: B. Milk the catheter tubing
C. Remove the catheter
A. Urinary control may be permanently lost to some D. Notify the physician
degree
B. Urinary drainage will be dependent on a urethral 481. The nurse would know that a post-TURP client
catheter for 24 hours understood his discharge teaching when he says I
C. Frequency and burning on urination will last while the should:
cystotomy tube is in place
D. His ability to perform sexually will be permanently A. Get out of bed into a chair for several hours daily
impaired B. Call the physician if my urinary stream decreases
C. Attempt to void every 3 hours when Im awake
477. The transurethral resection of the prostate is D. Avoid vigorous exercise for 6 months after surgery
performed on a client with BPH. Following surgery,
nursing care should include:

Fourth Year- BSN Page 49


500 Oncolgy Nursing Questionnaires

482. A patient with relapsing Hodgkins disease presents (D) Yolk sac carcinoma
with weight gain, foot ulcers, vision problems, elevated
blood sugar, oral candidiasis, and new onset of wildly 486. A 57-year-old man presents to his physician with a 4-
swinging mood changes. What is the most likely etiology of month history of worsening fatigue and generalized
this patients psychiatric symptoms? weakness. Further questioning reveals that his clothes fit
him more loosely now than they had in the past. Physical
(A) Adverse effects of bleomycin examination reveals generalized lymphadenopathy and
(B) Adverse effects of prednisone hepatosplenomegaly. Lymph node biopsy specimens are
(C) Adverse effects of vincristine sent to the pathologist with the presumptive diagnosis of
(D) Normal psychiatric response to having cancer lymphoma. Which of the following types of neoplastic cell
is most common in non-Hodgkins lymphoma?
483. A 41-year-old pregnant woman sees her obstetrician
because of new-onset vaginal bleeding. Although she is (A) B lymphocyte
only 4 months pregnant, her doctor notes that her uterus (B) Myeloblast
is the size usually seen at 6 months of gestation. Maternal (C) Plasma cell
blood works shows a -human chorionic gonadotropin (- (D) Reed-Sternberg cell
hCG) level >5 times the upper limit of normal. If left
untreated, what is a possible consequence of the patients
condition? 487.. A 29-year-old man presents to his primary care
physician with a painless testicular mass. Laboratory
(A) Choriocarcinoma studies show an elevated serum human chorionic
gonadotropin level. Which of the following is the most
(B) Coma
likely site of nodal metastasis in this tumor?
(C) Fetal neural tube defects
(D) Ovarian cancer (A) Deep inguinal lymph nodes
(B) External iliac lymph nodes
484. Oncology Nurse Test Questions about Hydrops fetalis (C) Gluteal lymph nodes
that occurs in the setting of a certain type of thalassemia. (D) Para-aortic lymph nodes
What is the underlying mechanism leading to this event?
488. A 28-year-old woman comes to the physician
concerned about an excessive amount of bleeding from her
(A) Excess -globin chains binding tighter to oxygen gums when she brushes her teeth. Her laboratory results
(B) Excess -globin chains binding weaker to oxygen show an increased partial thromboplastin time and an
(C) Excess -globin chains binding tighter to oxygen increased bleeding time, but are otherwise unremarkable.
(D) Excess gamma-globin chains binding tighter to oxygen Which of the following treatments will most likely alleviate
this patients symptoms?
485. A 56-year-old man who is a health care worker
(A) Cryoprecipitate
presents to his physician with vague abdominal (B) Factor VIII concentrate
discomfort. A physical examination reveals a tender liver, (C) Fresh frozen plasma( correction : vitamin K)
palpable to 6 cm below the costal margin and scleral (D) Low-molecular-weight heparin
icterus. His laboratory studies are significant for an
aspartate aminotransferase activity of 200 U/L and an 489. . Nurse Questions about a 29-year-old woman, who is
32 weeks pregnant and has been in the hospital for 3 days
alanine aminotransferase activity of 450 U/L. A CT scan of
because of pyelonephritis, starts oozing blood from her
the abdomen shows a dominant solid nodule in the liver. intravenous lines and bleeding from her gums. Petechiae
The marker most likely to be elevated in this patient is also are also noted in her skin. Laboratory tests show a platelet
a good indicator of which of the following malignancies? count of 98,000/mm3, hematocrit of 38%, WBC count of
8000/mm3, and a prolonged prothrombin time. What
(A) Choriocarcinoma other laboratory anomaly would also be expected?
(B) Colorectal carcinoma
(A) Elevated D-dimer levels
(C) Melanoma

Fourth Year- BSN Page 50


500 Oncolgy Nursing Questionnaires

(B) Elevated factor VII levels a. Associated with a 10% mortality.


(C) Elevated fi brinogen levels b. Contraindicated if 1 lesion is present.
(D) Elevated protein C levels c. Associated with 16-40% 5year survival.
d. Usually followed with consolidation
490. A 62-year-old woman presents to the clinic
complaining of frequent bleeding while brushing her teeth radiotherapy.
and easy bruising. She reports she recently had pneumonia 496. 2.Fluorouracil based chemotherapy:
and was treated with a broad-spectrum antibiotic. a. Is indicated for all patients with Dukes B
Laboratory tests show: Tumours.
Prothrombin time: 18 seconds b. In combination with capecitabine
Partial thromboplastin time: 37 seconds improves survival to around 18months in
Platelet count: 231,000/mm3
metastatic disease.
Hematocrit: 37%
WBC count: 4800/mm3 c. I s likely t impair quality of life when used
The cofactor that is deficient in this patient is needed for in the metastatic setting.
the carboxylation of glutamate residues of which of the d. Improves 5years survival rates for
following? patients with Dukes C Tumours by
approximately 7%.
(A) Factors II, VII, VIII, and X 497. . Which of the following is a feature of colorectal
(B) Factors VII, VIII, IX, and XII
cancer?
(C) Proteins C and S and factors IX, X, XI, and XII
(D) Proteins C and S, prothrombin, and factors VII, IX, and a. In DukeS 90% patients are cured by
X surgery alone.
b. Distant metastases are present in less
491. Risk factors associated with colorectal cancer than 5% of patients in presentation.
include which one of the following? c. Chemotherapy has no effect on survival.
a. Irritable bowel syndrome. d. In Dukes C disease <30% patients will
b. Low intake of dietary fiber. relapse within 5years.
c. Low intake of red meat.
d. Chronic aspirin therapy. 498. Bevacizumab:
492. Colorectal cancer:
a. Is a monoclonal antibody that targets
a. Is more common in males.
vascular endothelial growth factor
b. Is the most common cancer in the UK.
(VEGF).
c. Most commonly occurs in cecum.
b. Is a monoclonal antibody that targets
d. Commonly arises from pre-existing
epidermal growth receptor (EGFR).
adenomas.
c. Has no proven survival benefit in the
493. Characteristics presenting features of left sided
metastatic setting.
colorectal tumors include:
d. Is standard treatment for metastatic
a. Acute large bowel obstruction.
colorectal cancer in UK.
b. Iron deficiency anemia.
c. Painless abdominal mass. 499.. Regarding metastases from colorectal cancer
d. Foul smelling stools which are difficult to and symptomatic management of them:
flush.
494. Features associated with poor prognosis in a. Liver metastases are a rare feature.
colorectal cancer include: b. Hyperproteinaemia is common.
a. Adenocarcinoma cell type. c. Liver capsular pain usally responds well
b. Rectal bleeding at presentation. to non-steroidal anti-inflammatory drugs.
c. Presence of involved lymph nodes. d. Cerebral metastases are common.
d. Lymphocytic response of tumor.
495. . Surgery for liver metastases from colorectal 500 Acute bowel obstruction I advanced
cancer is: colorectal cancer:

Fourth Year- BSN Page 51


500 Oncolgy Nursing Questionnaires

a. Is usually managed by surgery.


b. Is usually caused by obstruction at a
single site in the bowel.
c. Should be treated with regular oral anti-
emetics, analgesics and anti-spasmodics.
d. Should be treated with using a syringe
driver containing a mixture of anti-
emetics, analgesics and anti-spasmodics.

Fourth Year- BSN Page 52

También podría gustarte